Collections: Total Generalship: Commanding Pre-Modern Armies, Part I: Reports

This week we’re going to start a three-part (I, II, IIIa, IIIb, IIIc) look at the role of the pre-modern general or army commander, particularly in the context of a pitched battle. This is of course a vast topic, but we are going to focus not on tactical or strategic questions but on a lot of the nuts and bolts constraints which condition those questions: where is the general, what can he know, what can he see, who can he communicate with and to what degree can they follow his commands effectively? After all, a plan which is perfect save that it cannot be communicated or executed isn’t perfect at all.

Pre-modern generals – and I am going to use this word in a very broad sense to mean the overall commander of an army, even in societies where that figure is a consul or king or what have you – figure very prominently in media. While modern warfare scenes in film are often shot from the perspective of regular soldiers, pre-modern battles are almost always shown at least partially from the perspective of the commanders: Théoden, Jon Snow, Alexander, Darius, William Wallace, Robert the Bruce, Henry V and so on. And when I teach I sometimes use these scenes to point out some of the absurdities: the commonplace of the general signalling with hands or a shout to an army obviously too large to see or hear him, for instance.

Generals are also the primary frame for pre-modern pitched battles in games, at least on any scale larger than a small skirmish. Of course there are games that are entirely about strategy or tactics, but even in fantasy or pre-modern RPGs it is most common for the player’s character to be put in command of the Big Battle than to merely be a foot soldier in it (or more common yet, to be a mere foot soldier in the Battle We Lost in the opening but in command by the end when it is time for the Decisive Battle To Decide the Fate of the World). As you may have gathered from the title, we’re going to use the Total War series of games predominantly to represent the common cultural model of generalship against which we can compare the actual practice of generalship.

It turns out commanding an army is easier if you can just have your body from the chest up hover like a horror movie apparition 50 feet tall in front of your army and shout orders at them. Alas, not an option for most pre-modern commanders.
Image is the menu splash screen from Total War: Rome Remastered

At the same time, as we’re going to see, these depictions – which for story reasons place strong emphasis on the control and agency the general supposedly has – for the most part fail to represent much of what a general can and cannot do in a pre-modern pitched battle, primarily by failing to take into account the physical and cultural constraints on the general’s actions. So we’re going to talk about those constraints and how they shape the job of generalship in a pre-modern army.

A note on scope before we begin: I am going to restrict this look to the broader Mediterranean in the pre-modern period. That means the Mediterranean littoral, plus the Middle East and Europe; many of these rules will apply elsewhere, but I am going to stick to where I know the sources. It also means the ‘pre-modern,’ by which I mean the period stretching from the end of pre-history (which varies place to place with the advent of written sources) to the beginning of the early modern period in c. 1450 or so. That distinction matters because technological changes after 1450 begin to alter some of these constraints, although generally slower than you might think.

And as always, if you like what you are reading here, please share it; if you really like it, you can support me on Patreon. And if you want updates whenever a new post appears, you can click below for email updates or follow me on twitter (@BretDevereaux) for updates as to new posts as well as my occasional ancient history, foreign policy or military history musings.

Total Tactics?

Fundamentally the tactical portion1 of Total War games (and indeed many real time strategy – RTS – games) is oriented around the concept of the ‘match-up. What I mean by this is that the main tactical puzzle the player-as-general is trying to solve is to achieve a series of favorable match-ups: tie up enemy cavalry with spearmen that perform better against cavalry while engaging enemy spearmen with swordsmen that perform better against infantry and so on. This is a simplification of course: these games often have special abilities, charge impacts and other considerations that complicate the basic match-up based puzzle; but the base of the puzzle is the match up (particularly in PVP matches where player-generals lack the easily abusable predictability of a computer opponent).

That shapes the way a Total War battle shakes out. To run through it very briefly, in the pre-battle phase, the player places their units in a formation, though I use this term loosely as high-level play tends to revolve around very loose ‘checkerboard’ formations with very large intervals between units. In practice the army doesn’t maneuver as a formation much in Total War games, but instead each unit (which tend to be around company-sized; c. 100 troops or so) maneuvers independently of all of the rest.2 The match then starts and one or both armies will close distance and what the player is looking to do is to make sure that their units end up in those favorable match-ups where their inherent statistics will enable them to do more damage (measured in unit value) than they take.

From Total War: Warhammer II, this is a ‘loose checkerboard’ of the sort that seems to be fairly common in high level competitive play (though I am sure I have messed up some of the finer details). The units are generally broad to command a lot of space and so that as many of their models get into the fight as quickly as possible. The wide spacing allows for maneuvers to get specific ‘match ups’ and also limits the danger of area-of-effect attacks (artillery and in this fantasy setting, spells). Heavy hitters (the greatswords) are towards the back of the checkerboard, so they can avoid enemy missile fire and be brought forward into their ideal match-ups, since their offensive power is important.
I know of no pre-gunpowder army that formed up like this, with such wide gaps between units, which in an actual battle would allow enemies to infiltrate the formation and tear it apart. Attempting to fight this way on an actual late medieval battlefield would be extremely foolish.

What exactly makes a favorable match-up varies. Many units in these games have specific attack and defense bonuses which only work when fighting specific kinds of other units (e.g. ‘bonus against infantry’ or ‘bonus against large/cavalry’). Units have armor values and armor-penetration values, so a high armor unit will ‘trade well’ against an enemy with low armor penetration because they’ll take very little damage. Slow units are vulnerable to missiles, but some units have shields which negate a percentage of missile damage and thus ‘soak up’ missile fire, making it a waste to shoot at them if any other targets are available. The term for a unit which is particularly effective against a particular kind of unit is a ‘counter’ (as in, ‘spearmen counter cavalry’); if the match-up is very lopsided so that no conditions could really shift its balance, it is a ‘hard-counter.’ Of course, there are instances where the way to a favorable match-up may be to double-up on a single enemy unit and some units benefit greatly from charging into enemies, encouraging ‘hammer-and-anvil’ tactics, but it is really striking how often these match-ups exist effectively in isolation, a point we’ll come back to later.

But for now what I want to focus on is what that makes the player’s task of generalship about. Typically in a Total War game, once the battle begins, the player has nearly perfect information about an enemy formation. Terrain can create exceptions, but like most strategy games the Total War tactical information environment is binary: either you have no idea what enemy units are in an area, or you can see exactly what units are present: how many of them, their formation, what type of units they are, their position, facing and motion, their exact combat stats and often even how much damage they’ve sustained or even how tired they are instantly at a glance.

And here, also from Total War: Warhammer II is a ‘tight checkerboard’ of the sort one sees a lot in high level non-competitive play against the AI (e.g. campaign on higher difficulties). Unlike the competitive formation, I’m a lot more familiar with these as I do play at higher campaign difficulties. Winning at these difficulties is, for most factions, a question of getting the most work out of ranged units (since the difficulty bonuses the AI receives means that your melee units will almost always lose all but the most favorable match-ups) and that’s what this formation is designed to do. The checker-shape allows for easy firing lines, but the gaps are too small for the AI to effectively infiltrate to the ranged units in the back. The great advantage here is thus that units in the interior of the checkerboard can keep firing even if units on the outer edges have been caught in melee, forcing the AI to rout the entire thing, one unit at a time, while getting pummeled by ranged fire (note how we went from 6 ranged units in the PVP army – and that might be too many – to 9 here, and honestly that isn’t nearly enough; ranged units are far more heavily used in single-player campaign).
Visually this army formation resembles the Roman quincunx, but it functions nothing like an actual Roman quincunx (which would have formed a solid line to actually fight). Instead, it actually fills a vaguely similar role to a pike-and-shot formation like a tercio, a formation which, as we’ve noted, is missing from the Total War series.

At the same time, players can issue immediate orders to their units, allowing rapid responses to new information. While units may still take time to move, they can be gotten moving almost instantly, so a player that sees the possibility of a favorable match-up (or the danger of an unfavorable one) can immediately begin shifting units around within the army to try to get the advantage. Consequently, the player’s main task is to quickly ascertain the enemy units position and type and then maneuver their own units as they (or the enemy or both) advance so that the match-ups (and to a lesser extent, positioning) that result are favorable.

And what is striking to me (and thus forms the basis of this series) is that this main task has functionally nothing to do with how ancient generalship actually functioned. This basic task of the most popular historical real-time strategy game series almost3 never shows up in the sources. Indeed for reasons that, as we proceed, will become clearer and clearer, it was impossible for most generals in most battles in the ancient world to do this once the battle had begun. It was more possible, as an aside, in early modern warfare but that is a period these games rarely simulate and indeed haven’t really returned to in earnest for more than a decade.

Instead what we’re going to see is that command in the ancient world (and indeed, in any environment) existed under a set of fundamental constraints which historical strategy games generally remove for ‘playability’ reasons. And I should be clear here: I understand why in many cases developers do this. The goal of these games is to be fun and it is seemingly axiomatic that ‘fun’ results from player empowerment and as we’ll see all of these constraints are profoundly disempowering in different ways. But the removal of those constraints tends to mean that even with historically inspired units and armies, the actual tactics and battles in these games don’t much resemble their historical inspirations because skilled players rely heavily on tactics and maneuvers which the historical inspirations for these armies weren’t capable of.

The same is true in different ways for film depictions of generalship. There are exceptions, of course, but in most films where an ancient or medieval general is prominent, the film-maker is either developing or revealing that character through a battle sequence, which in turn generally requires perceivable cause and effect between the general’s actions and the battle’s outcome. Moreover the need to build tension typically means that crucial decisions are delayed to create that moment where the heroic leader shouts ‘charge!’ or ‘fire!’ at just the right moment to win the day. Rarely does one get a battle scene in which all of the decisions of import were made at a council of war in the early, pre-dawn hours with the battle then left to simply play out according to plan and largely out of control of the participants.4 Instead characters in command make decisions on the fly which often could not have been done without planning.

Still from HBO’s Rome, the Battle of Philippi. As I noted above in a footnote, this battle depiction is actually rather weak, but this one scene is great. Octavian asks Antony, “What is happening? Do you know?” and Antony responds, “No Idea.”
In the actual battle, the armies engaged – around 100,000 on each side – were so large that command was divided, with Antony and Octavian each taking a wing of the army (and Brutus and Cassius doing the same), meaning that on the first day there were effectively two disconnected battles occurring in parallel. Most pre-modern battles were not so large, but the limitations of communication (discussed next time) often meant that subordinate officers had considerable command independence.

We are going to look at those constraints in three parts. This week, we’ll look at the information environment that an ancient or medieval general had access to: what could they know and how could they know it? Then next week we’ll look at the ability of those same commanders to use that information to coordinate their army. Then in the third part, we’ll look at the ability of ancient armies to act on those commands effectively and quickly, as well as suggesting an overall battle model that might be more akin to how an actual battle was commanded. Finally, in a fourth bonus part, I want to look at a number of the other tasks generals were expected to do which are typically not featured (or at least not mechanically simulated) in these sorts of games much at all.

Reports

One of the fundamental structuring assumptions of the ‘match-up’ model of generalship is the ability of the general to adequately observe the enemy force in order to identify both its component units and their positions. In a Total War game, you an generally see enemy units quite easily and if you mouse over them a helpful tooltip will inform you exactly what kind of unit they are, often down to their precise statistics. Of course real battlefields do not come with zooming cameras or tooltips. So what kind of information might an ancient or medieval army commander have on the day of the battle?

Battles, of course, do not spring up out of nowhere but are generally the result of a campaign. Typically the attacking army’s goal was to ‘deliver a siege‘ to a key enemy population center as a way of gaining control over the surrounding territory (and possibly convincing other, smaller population centers, even fortified ones, to surrender preemptively). There are exceptions, of course; ancient Greek hoplite armies had such limited siege capabilities that they often didn’t attempt to deliver the siege at all but instead threatened agricultural disruption in order to force a battle. In any event, when both armies arrive at the battle it is because they have been maneuvering for some time, during which one might reasonably expect both commanders to be trying to get some sense of the other army’s strength, position and intentions.

Generally the commanders are going to have to rely on a broad category of intelligence we might collectively term ‘reports.’ Some of those reports might be from detachments (generally fast moving cavalry) from their own army on intentional scouting missions, but just as often the ‘reports’ were generated by civilians. Traders moving along the roads might be asked for information or rumors; friendly civilians in enemy territory (frequently of a political faction currently out of power that might hope for a change in their fortunes) might also send word of an army’s movement, its size or intentions. And of course as elements of the army passed through smaller population centers, they could collect whatever rumors were available. Of course one’s own scouts were the most reliable, but in order to be able to coordinate their movements with the army and send messages to it, scouts tended not to stray too far from the army, so information on the situation in regions where the army wasn’t often could only be had through less reliable channels.

Games struggle to reflect this sort of information environment. Often games present a ‘fog of war’ of sorts – areas where the player can see the terrain, but not the movement of enemy units – but the real fog of war in this kind of warfare was a lot more complex. First of all, the general didn’t know where it was. A lack of reports from a given region might mean it was quiet, but it might also mean that, for whatever reason, the general is blind to events there. Moreover, in most games what information the player does have is invariably accurate, but the very nature of reports is that they are very frequently wrong. Indeed, intentionally seeding false reports was a standard stratagem.5 The general in campaign is thus not looking at an incomplete but secure picture of the war on a map, but rather trying to make sense of a cloud of verbal or written reports (delivered as words, with all of their shades of meaning and difficulties of interpretation) many of which are inaccurate and some of which are blatantly false.

Reports of these sort also time time to travel. As I remind my students, from antiquity to the early 1800s, you are dealing with societies where the most sophisticated communications technology generally available was ‘man on horse.’ Now a messenger rider that is regularly changing horses at pre-appointed stations can move very fast indeed; in excess of a hundred miles a day. But reports are only going to move at that kind of speed when carried by official messengers on pre-set routes; generals in the field will rarely have use of that kind of message system. Instead, reports are going to move at the speed of merchants or the speed of rumor, which is going to be much slower, far closer to the movement speed of the enemy army. This leads to situations where generals often know what general region an enemy force is in and they may have a sense of its ultimate objective (because the report they have was that the army was in such-and-such town moving down such-and-such road ten days ago) but often not its exact position at any given time. At the same time, again, no reports doesn’t mean there’s no army somewhere; it just means there are no reports! Unlike in most strategy games, a pre-modern commander doesn’t really know where the fog of war is or what might be hiding in it.

For instance, one not-uncommon occurrence in ancient battles was the appearance of some detachment of an army being interpreted incorrectly as an entire second army appearing. For instance at the Battle of Delium (424 BC), the Athenian right-wing, having defeated the Boeotian forces opposite it, panicked at the sight of a relatively modest detachment of Boeotian cavalry sent by Pagondas (the Boeotian general) in a desperate effort to shore up his collapsing left wing; the Athenians thought this was an entire fresh army (Thuc. 4.96.4) and routed. Likewise, at the Battle of Tifernum (297) between the Romans and the Samnites (Liv. 10.14), the Roman consul, Fabius Maximus Rullianus panicked the Samnite army by having a small detachment of his infantry march around the battlefield ahead of time and reveal themselves only after the battle was joined, appearing like a freshly arriving army. What Fabius was counting on here is that there was another Roman army in the field (under the command of the other consul, Publius Decius Mus) but it was too far away to assist in the battle; but of course the enemy (and indeed, Fabius’ own soldiers) didn’t know that so the sudden appearance of the ‘advance guard’ of Decius’ army might inspire panic in the enemy and resolve in the Romans, which it did.

The other side of this problem is the ‘encounter battle’ or ‘meeting engagement,’ where two armies unaware that they are in close proximity run into each other. One such example, famously, was the Battle of Cynocephalae (197) between a Macedonian army under Philip V and a Roman army commanded by Titus Quinctius Flamininus. Neither Philip nor Flamininus knew exactly where the enemy army was, but both were working on reports about their general location (Plb. 18.18-21 which is also a great example of two armies attempting to maneuver operationally against each other for advantage while relying on reports) and so knew the enemy was fairly close.6 Thus both generals dispatched scouting forces to seize a large hill (called “the Dog’s Heads” or Cynocephalae) for observation and as covering forces; the scouting forces collided and when both commanders attempted to reinforce them (based on the reports those engaged scouts sent back) it precipitated a general engagement, which the Romans won. Of course it is striking that this is how things go when careful scouting is done; the alternative were the relatively rare (far rarer in history than in Total War games) true ambush battles like Lake Trasimene (217).

I can think of few games that model this kind of information environment. Perhaps the closest I’m aware of are Radio Commander where you have to rely on the spoken reports of forces in the field and HighFleet which forces the player to rely on a mix of signals intelligence (SIGINT) from intercepted transmissions to both radar and systems to detect enemy radar, systems which often provide incomplete or confusing information (“there’s something over there, but what is it and how many?”). That said, this information is generally correct, merely incomplete and in any case is instant whereas the information available to a pre-modern commander was often simply wrong or days or even weeks old by the time it was received.

All that said, one thing various reports might well inform a general is of the relative size and composition of his enemy, at least in the most general terms. That was a sort of information which was, at the degree of accuracy we are dealing with, less time sensitive. The mustering of a large army, after all, was a hard thing to fully conceal, as was its movement. Observers might well note specific kinds of soldiers and very roughly their numbers. This sort of information was never so precise as it tends to be in games, where the player often has an exact enemy order of battle at time of contact. But it is clear that generals often knew both when they substantially outnumbered their foe and when the reverse was true and could act accordingly. Especially once armies drew close, any capable scout could size up the camp and the number of campfires (although faking these signs was a standard stratagem too).

Offering Battle

Moreover, while the precise location of an enemy army might be a mystery, its general location was often known and as a result, true encounter battles in the pre-modern world were the exception rather than the rule. Armies had the move have to forage for supplies, meaning that the presence of an enemy army in the neighborhood is news that is likely to travel fast, so two armies are unlikely to be very close to each other without knowing it.7 And as noted, armies tended to move with scouts, creating a ‘bubble’ of more accurate information which made accidentally running into an enemy army unusual. Unlike in modern warfare where just about any enemy that can see you can shoot you, we are dealing with armies whose weapons have effective ranges far shorter than human visual range, so in the absence of some sort of obstruction (like the hill at Cynocephelae) scouts are likely to observe each other long before they crash into each other and thus be free to report back on the presence of the enemy.

At this point the question is offering and accepting battle; I want to note that I am going to use the Battle of Pydna (168) as an example a few times here so I want to recommend for a second time the recent and excellent reconstruction of the battle published by Paul Johstono and Michael J. Taylor in GRBS. This is, to modern thinking, such an odd affair that it deserves some explaining; it is often missed even by modern scholars but is a commonplace of pre-modern battle and campaign narratives or mistaken for ritualism when it is just good sense. Armies, of course, do not march in fighting formation; to fight an army generally wants to form in a wide line, but roads are narrow, after all. No general relishes the idea of throwing soldiers straight into battle exhausted after a whole day of marching either, so the armies are likely to encamp near each other with the plan to offer battle on the following day. You may reasonably ask why one army doesn’t skip this step and roll straight in; this happens but generally rarely. Generals tend to encamp their armies in either fortified camps, on solid defensive positions or, where possible, both. So the attacker, exhausted from marching, would have to open the battle with an assault on well-defended position; hardly an appealing option in most cases.

So frequently final preparations for a battle were made with both armies encamped fairly close to each other, often separated by just a few miles of open field. Even if no battle occurred immediately the following day, armies were not idle: scouts and pickets would be set out to give advance warning of enemy movements or an attack on the camp, while foraging parties both food and water might also be dispatched (more on these a bit later). For logistics reasons (again, a bit later) both armies encamped this way are on a timer before their local supplies run out and they must move, so they cannot sit this way indefinitely (which both generals know about the other). At some point they must either fight or withdraw.

Thus the other thing an army in this situation might do: offer battle. One (or both) armies might draw up their fully array, in battle formation in front of their camp, essentially signalling “I’m up for a fight if you are.” Now of course generals are likely to form up on favorable terrain (which might mean the heights or areas where the width of the battlefield is favorable) and don’t like to fight on terrain favorable to the enemy, so you might end up in a situation where, over multiple days, each army forms up on terrain that is good for them and then refuses to fight on the other army’s ground. This happens for instance at the Battle of Mantinea (418) between Sparta (and friends) and Argos (and friends); the Argives formed up on a solid position and while the Spartans initially prepared to attack the king (Agis II) decided at the last minute not to and withdrew, refusing battle; the Spartans subsequently lured the Argives into less favorable terrain by damaging the irrigation system, forcing a battle on their terms, which the Spartans won (Thuc. 5.64ff). Likewise, at Pydna (168), both Roman and Macedonian armies meet on a narrow plain near Pydna, but the Romans – fresh from a long march and probably preferring a wider battlefield – refuse battle and withdraw, leading to fairly wild battle that occurs the following day. At Philippi (42), the two armies (Octavian and Antony leading the Caesarian force on one side and Brutus and Cassius the Liberatores on the other) encamped opposite each other for a number of days (we are not privy to the exact number) while Antony offered and indeed tried to force a battle. He got his battle on October 3, but it was indecisive, leading to another period of waiting, offers of battle and refusal culminating in a second engagement on October 23, by which point the armies had probably been camped within a few miles of each other for at least a month.

Offering battle gives the general in command some options in terms of time. He can, for instance, choose what time of day to offer battle, with the classic stratagem here being that a general who thought a battle risky might only offer battle late in the day, so that if the battle went poorly the enemy pursuit would quickly be ended by nightfall. Of course the reverse is just as true: a commander who is very confident and hoping for a decisive victory might prefer to commence the battle at dawn. He can also try to choose the ground; if cautious a general can form up on the presumably favorable ground near his camp. By contrast, offering battle in the low ground between the two camps8 was a sign of boldness and confidence. There might thus be a kind of negotiation of a sort; no one is passing notes back and forth, but each general can gauge the other’s intentions from how their army forms up. And of course a general who sees his opponent forming up on favorable ground near their camp, but who is supremely confident of his army (or rushed by a lack of supplies) might attack anyway, accepting battle on his opponent’s terms.

But to understand the information a general is working with when he makes the decision to offer battle, we have to think about what the process of actually forming an army into its battle array involves. In strategy games, this is generally an easy process: you click and units move exactly where you clicked. There is no confusion, no one gets lost, everyone understands your orders. But that isn’t how directing real groups of humans works in the real world. One exercise I do with my warfare classes is to form up the front row of students to make up a single half-file of a Macedonian sarisa-phalanx; without fail one needs to budget several minutes for this fairly basic task of “form a line one arm’s length apart facing this way.” The issue gets even more complex, of course, as the general is dealing with groups so large they cannot all hear his orders at once and in many smaller units each of which needs to know where it is going. In short, forming into battle array cannot be done ‘on the fly,’ this is a fairly complex, difficult task which takes time and planning.

So how was it done?

Generally plans for a battle, if any planning was to be done (we’ll talk about how armies responded when detailed planning wasn’t possible in the third part of this series) were drawn up the evening (sometimes in the very early morning) before in a ‘council of war’ where the commanding general met with his subordinates and advisors. Livy notes just such a council on the eve of the Battle of Pydna, for instance (Liv. 44.36.7-14) and it is very clear that such meetings were standard practice; the Latin term for all of the various people who ought to be included in such a council was the general’s consilium. Such a council provided an opportunity for the general to work out the details of exactly how they wanted to conduct the battle the following day so that all of their subordinate officers knew what to do. My own sense of descriptions of these councils in the ancient sources is that they were more often consensus-building tools rather than deliberative ones: the goal was to get everyone on the same page with the general’s original plan, not to submit the plan to committee.

The army then has to be gotten into formation. Since it is exiting a camp (which is going to have a narrow, road-width entrance) this entails reforming from a marching column leaving the camp into a battle line. The expedient to make this easier was to have a fighting depth (the number of men in a file) either equal to or in multiples of your marching width, so that a unit could exit the camp in column, turn either right or left, march to the appointed place and then everyone could simply right-face or left-face (that is, a 90-degree turn) and thus be in fighting formation. The relative position of units then, and thus the organization of the overall line of battle, would be determined by the order that units exited the camp. So at that council of war the evening or morning before the battle, the formation can be decided on and the order of march out of the camp into that formation set. All of that is going to make getting a huge body of thousands of men out of their tents, into their equipment and then into the right positions in the battle array possible in a reasonable amount of time.

But notice what the general does not know in that moment. He does not know if the enemy intends to accept his offer to battle, of course. He does know where their camp is, so he knows in what direction they will approach from, but he does not know what formation they will adopt, nor do they know what formation he will adopt. Because forming up for battle takes a while, it was generally impossible to wait to see what the enemy’s formation was before adopting your own: you’d be very vulnerable indeed if the enemy, fully formed, simply advanced on you while you were only beginning to form up (there having been a delay to plan your marching order and thus formation). So the basic question of the overall formation has to be made effectively blind to the enemy’s formation. The general might know their numbers, direction and even some of their composition (things like “the enemy has brought many war elephants” or “their army is much larger than mine”) but probably not their formation.

And this goes a long way to explaining why armies in the ancient or medieval world tend to have ‘standard’ battle formations. They cannot respond to specific enemy dispositions in advance anyway, so armies tend to form up in predictable ‘safe’ patterns. Typically that means the heaviest, least mobile forces (generally heavy infantry) are deployed in the center, with progressively lighter and thus more mobile units on their wings, where faster maneuver might be necessary. The planning at the council of war could take terrain into account; it was very common for armies to seek to ‘anchor’ one or both flanks on terrain unsuitable for fighting (rivers, marshes, hills, dense forest) in order to create a controllable battle-space or to deploy light infantry in difficult terrain where heavier infantry or cavalry might struggle.

Of course a general might well guess at this point how an enemy will form up over the terrain and seek to prepare for that, although they are only guessing. At the Battle of Magnesia (190), the Roman commander, L. Cornelius Scipio (the brother of Scipio Africanus) gambled that a constraining river on his left would mean that his opponent, Antiochus III (king of the Seleucid Empire) wouldn’t launch a heavy attack here (Liv. 37.39; App. Syr. 31). Consequently, he left only a small force on that side in favor of firming up his center and right. Scipio, by no means a poor commander, was in the event wrong in his gamble: Antiochus did end up placing his elite cavalry (the agema, the king’s cavalry bodyguard) on that side and broke through the thin forces Scipio had placed there, though the battle was salvaged by the Romans and their allies9 being victorious on the other side of the field and Antiochus’ inability to keep his victorious right from trying (and failing) to raid the Roman camp.10

Sometimes those formations were culturally loaded too; Greek hoplite armies, which were almost always composed of a number of allied states fighting together, generally deployed their hoplites in a single line (of variable depth, generally around 8), but position in the line signaled the importance and honor of each alliance member: the far-right was the position of highest honor, the far left the next-most important position, and then each other position was essentially ranked from right to left. Consequently the formation of hoplites might be as much about diplomatic ‘alliance maintenance’ and civic pride as it was about tactical expediency, which was part of by the Theban general Epaminondas’ decision to load up all of his elite Theban troops on the left wing at the Battle of Leuctra (371) was, in the Greek context, so innovative (this resulted in the Thebans crashing directly into the main Spartan contingent, which was Epaminondas’ goal).

Mark One

Once the two armies have begun forming up, our general is going to be getting information a bit more directly. In an environment where the very longest ranged battlefield weapons had maximum ranges of a few hundred meters and effective ranges under 100m, armies can form up quite close to each other, enabling the general to potentially see quite a lot without worry that his position is immediately dangerous. And most pre-modern generals will have one key visibility advantage at this juncture: they’re on a horse (or a chariot, or an elephant), giving them an elevated observation position that allows them to see over the large bodies of infantry moving around. On the other hand, the general’s ability to see is restricted to where he is; he can move around but not infinitely fast (as we’ll see in the next part, that’s also an issue for giving orders).

Now I am sure some readers are eager to note that a number of games have taken this particular limitation into account! Some Total War games had difficulty settings which restricted the camera to be within a certain distance of either your general or friendly units. Take Command – 2nd Manassas (2006), a game we’ll have cause to revisit more than once, likewise restricted the camera (based on rank). And the Mount & Blade series (we’ll focus here and subsequently on the latest entry, M&B 2 Bannerlord) does this aggressively as well. But even these examples fudge the issue in two important respects: height and distance. Because our general doesn’t have an aerial camera or a zoom feature: he is working with the Mark One Eyeball (henceforth abbreviated as Mk1 Eyeball).

Height is easy to talk about in this context. As noted a general is generally going to have a modest visibility advantage by virtue of being on a horse, but games in third person generally place the camera even higher. Mount & Blade‘s camera generally hovers a few feet above and behind the player’s character, for instance, while Take Command and Total War both position the camera much higher even if its position remains restricted. That matters a lot of course both because obstructions (trees, fences, hills) tend to occur close to the ground and also because the higher up a point of view is, the longer its horizon becomes. But the Mk1 Eyeball is, frustratingly, stuck at eye-level.

Screenshot of mine from Mount and Blade 2: Bannerlord. Note the camera is not merely behind the player-character (the mounted character in the bottom center of the screen) but also several feet above them. This is also, I should note, by the standards of M&B2Bannerlord a fairly large army with a few hundred soldiers (the infantry wall you see in the center is a square – not hollow – and the center of the archer line is just behind the center of the square).
As an aside, this formation is my own ‘budget tercio’ formation I frequently use to quickly draw up my infantry-heavy armies (I prefer infantry-centric armies), especially against horse-archer heavy opponents. You can formation-switch the archers from loose to line to get them bunched up behind the square if they’re in danger of being overrun by cavalry.

More complex but also important is distance. In Mount & Blade, a very large royal army might have a few thousand soldiers in it, of whom only perhaps 500 or so (depending on how powerful your computer is) are in the battlefield at once. Total War battles are larger, but a ‘full stack’ (a maximum size army) is typically 20 units of around company size (c. 100 soldiers) each; depending on faction and game that tends to come to a bit less than 2,000 soldiers once units with far fewer troops-per-unit (cavalry, artillery, etc.) are accounted for. As a rule, recent Total War games have tended to cap the number of armies that can engage at once in a battle to two full stacks, so 40 units and typically quite a bit less than 4,000 soldiers (and indeed, large armies like this are so cumbersome and difficult to control that it is no wonder that Creative Assembly, the developers, haven’t tried to use improved computer power to make the armies any bigger).

But armies in large pitched battles are often much larger. The very largest pre-modern armies in the broader Mediterranean tend to be in the 80,000-100,000 range (Cannae, Magnesia, Philippi) though these are rare. Nevertheless, armies in the 10-40,000 range were relatively common for large forces expecting a major pitched battle (note that armies in the middle ages tend to be on the smaller end, whereas armies in antiquity will tend to be on the larger end). The standard Roman consular army in the Middle Republic was a pair of legions (4,800 men each) matched by a pair of socii alae, for a total army size just short of 20,000; this was effectively the floor for a major Roman army, since non-Italian allies or more legions might well be added to this if necessary. The total Roman force at Pydna was fairly typical at around 35,000 all told, facing perhaps 30-40,000 soldiers, both Macedonians and their allies. Medieval armies were often somewhat smaller; the English at Agincourt only had around 8,000 troops (to the French’s perhaps 15,000), though of course the size of a cross-channel expeditionary force was naturally constrained.

Bringing back this picture from Total War: Warhammer II, there are just 1,360 soldiers in this image; this would thus be a very small army for a pitched battle, the size of a mid-sized polis army (like Plataea, for instance) or the personal retinue of a high but not-royal noble in the Middle Ages. A single tercio (of which an army might be composed of several) would be more than twice this size at paper strength; a Roman legion more than three and a half times the size. Alexander’s heavy infantry line at Issus alone was more then seventeen times this size (his whole army around 27 times this size).

Now I should note here that of course there were a lot of engagements which were much smaller than this. In the ancient world there were still skirmishes between scouts and foraging parties which would involve only small detachments of these large armies. And in the Middle Ages, a lot of the endemic warfare was between relatively small forces in wars between more minor nobles. Of course, apart from Mount and Blade, that kind of lower-scale endemic warfare is something games generally do not simulate at all (and films generally only feature by accident when they fail to put enough people on screen for what are supposed to be ‘big’ battles). So we are going to remain focused on large-scale pitched battles here.

And the thing is, an army of 15,000 or 35,000 is a lot bigger than an army of 1,500. And I don’t just mean in terms of its numbers; it is physically bigger – it occupies more space. And here I can use Johstono and Taylor’s Pydna reconstruction (linked above) to make the point, aided by google streetview, because they’ve calculated and figured the likely positions of the various elements of the battle. Now of course they present the fight in a nice, neat map overlaid on the modern topography using Google Maps; these maps are lovely but of course they are the battle as seen by aliens from space not as seen from the ground. But thanks to the wonder of Google Street View, we can get a vague sense of what you might have been able to see from where, aided by the fact that, being mounted atop cars, the street view camera isn’t far from the height of a man on a horse.

Some brief background on the battle and the relative positions. The Macedonian army, under the command of their king, Perseus11 had occupied a blocking position on the road leading up to Pydna and through it into Macedonia proper. The Romans had arrived the previous day but their general, Lucius Aemilius Paullus, had refused an engagement on ground so favorable to the Macedonians (who had a sweet, narrow little position at the northern edge of the field) and had instead gone into camp a few miles to the south. Thus, after the two armies had encamped opposite each other, neither had accepted the offer of battle on the other’s terrain. However, they both had the same water source, a stream on the eastern edge of the battlefield. Thus, the day after the Romans arrived, parties from both armies going to get water ended up in a small scrap which, as both armies rushed to reinforce them, precipitated a general engagement.12 Thus each element of both armies came into action one by one, beginning on in the East and moving West as the two armies marshaled out of camp (generally very close, as Plutarch notes, to the Roman camp, in part because of where the skirmish started, but also because the Romans were initially pushed back). The Roman army was fundamentally in four roughly equal-sized major units (from left to right or west to east): the left wing (Sinistra Ala) allies, then the second legion (Legio II) then the first legion (Legio I, where Aemilius, the Roman commander was), then the right wing (Dextera Ala) of allies.

Plutarch (Livy’s account of this battle has a large gap that sadly covers most of the action) has a passage in this context about the fear that Aemilius experienced when he saw the Macedonian heavy infantry, the chalkaspides (literally ‘bronze shields’ – part of the Macedonian sarisa-phalanx which wielded the 21ft sarisa pike along with a shield held by a strap grip; the shields were not entirely bronze but faced in it, in contrast to the Leukaspides, presumably less elite troops whose shields were probably faced with hide, painted white) – advancing (Plut. Aem. 19.1-3). To get there, Aemilius will have led Legio I out of his camp, fortunately (for us) advancing directly over a road with street view, so we can get a sense of what he’s seeing:

Image via Google Maps Street view. The crest with the blue line across it would have been the rough position of the Macedonian Chalkaspides as Aemilius’ Legio I was formed up to the right of this vantage here. In the chaotic battle, these were some of the last moments for Aemilius to make any kind of maneuver decisions and you can see the information he’d have to work with would have been slight indeed (in practice Plutarch tells us that by the time Aemilius had a good sense of the battle, his heavy infantry was already engaging the Chalkapsides, putting the course of events mostly out of his control).

The blue line there just coming over the opposite hill crest some 550m away is about where Aemilius’ screening force initially had been and where the chalkaspides were advancing rapidly; it is striking that the thing Aemilius can apparently see is the bronze-faced shields of the chalkaspides which makes a lot of sense: at this distance he can hardly have seen much else, but he will have known when the men in the phalanx readied their shields and lowered their sarisae because of the gleam of the shields (which is, by the by, exactly what Plutarch describes him as being able to see: that the Macedonians had readied their shields). Nevertheless it is only as his legion is beginning the attack that Aemilius is fully aware of what unit is directly in front of him and that it is advancing in good order (his subordinate, Scipio Nasica, forward with the screening force, seems to have had a better view, Plut. Aem. 18.5-8, and may well have rushed back to tell him – those reports again). It is, by this point, probably too late to send Legio I anywhere else; Aemilius seems to have concluded that on his right a detachment of allies Paeligni and Marrucini could keep his flank secure (he was right) and so he best plow forward into the oncoming chalkaspides head on.

But that is just Legio I, which, by the by, at 4,800 men will have filled the entire width of that shot above as they advanced (imagine them about where the one brown tree is halfway down on the right, but extending all the way from the left to the right and a little off the picture on both sides). What about his right wing? Well, it’s just on the other side of that building (which obviously wouldn’t have been there in 168)!

Oh, you can’t see the building? Let me zoom in. It’s on the other side of THIS building.

The one that is simple a white spec in the original image. That would be the closest flank of the dextera ala, which would probably extend another 500m or so beyond that. Of course Aemilius’ Mk1 Eyeball did not have a zoom feature; a battlefield-useful telescope was at least 1500 years away. At this distance, Aemilius might have actually been able to make out the war elephants deployed on that flank (Liv. 44.41.4), but not much else save that his far right flank was still there, if he could see the glint of their helmets over the crest of the hill. What about his left flank? Well, here there’s a problem: there’s a hill in the way. Not a very big hill but hills don’t need to be very big to be taller than a man on horseback at their foot. Still, if we jog about 400m down the road we can get a decent impression of the left flank’s view from the ground:

The circled building there is, in Johstono and Taylor’s reconstruction where the right edge of the Sinistra Ala and the left edge of Legio II will have come together. Where this picture is, we’re probably 200m or so closer than the center of Legio I would have been, but perhaps Aemilius could have galloped over here to get a quick view of how L. Postumius Albinus, in charge of his second legion, was managing. He can hardly have seen much except that they were still there and presumably holding their ground and even less of the left wing of his army – the closest edge of which was still 800m away from the far leftward edge of Legio I. So these images are essentially the views that Aemilius could get, assuming he had nothing else to do except gallop from one vantage to another trying to figure out what was going on (in practice we are told that instead he seems to have stayed with his Legio I and rallied them in the advance; Plut. Aem. 19.3. Doubtless this was a better use of his limited time).

Because here’s the thing: a Total War army (single, full-stack) is going to occupy a frontage in a game of usually around a few hundred meters. Whereas, as reconstructed by Johstono and Taylor, the Battle of Pydna – by no means the largest battle the Romans ever fought – takes place on a field perhaps 2.5 kilometers wide (a bit wider than it might otherwise have been because of the chaotic situation and the gaps). Not every battlefield was so sprawling, but even small battles might be quite wide. The Athenian/Plataean army at Marathon was just 10,000 hoplites, deployed eight deep except for 2 of the 10 Athenian tribes in the center which were just four deep, which with a little math suggests a front rank roughly13 1,475 hoplites wide. The spacing in hoplite warfare varied, from synaspismos (tight formation, literally ‘shields together’) probably giving each man about 1m of space (and overlapping the shields) to a looser spacing perhaps twice as much.14 So that combined Athenian-Plataean army, despite being only 10,000 hoplites, might have been anywhere from 1.5 to 3km wide, likely to the narrower side given the relatively small size of the plain in most places.

There are some places on Earth flat enough to allow a man on horseback to see the c. 3.8 miles to the horizon, but even in those few, exceptionally flat places, formations of men more than a mile distant are going to be barely visible blobs moving over the landscape. Enemy formations are going to be harder to see and further away. Video games don’t work on this scale and frankly film rarely does either; when a large army is shown on film, they are generally arrayed in such a way to allow the camera to capture all of them, but real armies do not form up with the aim of easy observation by the enemy.

It is striking, as an aside, that these games tends to simulate the command of an entire army by giving the player the task of commanding forces that are generally around the size of a standard maneuver unit15 like Roman cohort of the Late Republic (c. 480 men and about the max number of deployed soldiers on default settings in Mount and Blade 2) or a American Civil War brigade (c. 1000-1500 and about the size of an army in a Total War game). Actual armies, of course, would be composed of many of these maneuver units; a Roman legion is ten cohorts and a Roman army, as noted, consists of multiple legions.

Instead with actual army sizes, a general is limited in his ability to observe the battle mostly to the rough positions of the component parts of his army and the enemy’s army; he may not know where specific units are unless they are very distinctive because most of them are at distances where he might struggle to make out specific equipment (uniforms and battle standards might clear this up a bit, but by the time we’re getting armies in uniform ‘facing colors‘ we’re well into the early modern period). Good observation positions (ridges, hills) might enable longer sight-lines, but they’re not going to enable more detail because again, all anyone has is the Mk. 1 Eyeball. This is why the general with his headquarters on the hill behind the battlefield is much more of an early modern or modern thing: telescopes and binoculars enabled generals to observe much more of the battlefield if they could get a good vantage.16

Seeing the Battle

And all of this is before the general confusion of battle makes a mess of things. Without standard uniforms (and often even with them) even from a moderate distance making out one block of troops from the next is difficult; friends mistaken for foes and vice versa even within musket range is hardly an uncommon occurrence, for instance. More broadly, depending on the terrain, thousands of feet moving at once can kick up a lot of dust, further obscuring the field.

From Alexander (2004), the Battle of Gaugamela. This battle scene has a few issues but is in general an uncommonly good representation. I particularly light how the dust is handled; fighting on this kind of terrain would throw up a lot of dust, making it hard to see much of anything. Of course in practice that will have mattered little to Alexander, who had left Parmenion in command of the phalanx, something we’ll come back to next time.

The upshot of all of this is that at each stage of the fight – the march to the battle, the night before, the forming of the battle array and then the battle itself – the information environment the general is forced to operate in is heavily constrained. In particular it is constrained in ways that make a lot of Total War generalship (and a lot of Hollywood generalship) at best deeply uncertain and more often effectively impossible.

At best if a general wants to ensure specific ‘match-ups,’ he is going to have to largely guess where the enemy might deploy those forces and arrange his both battle array – well before he can know the enemy’s disposition – to try to get those match-ups. Doing this was possible (as noted Epaminondas does this at Leuktra; Hannibal’s plan at Cannae likewise required guessing the Roman plan in advance), but tricky guess-work rather than something that could be improvised on the fly.

Think back, for instance, to the example from Pydna. By the time Aemilius can actually know for certain the formation his opponents had adopted (which was, in any event, somewhat unusual in that the presumably more elite chalkaspides were on the center-left of Perseus’ line rather than the center-right; it seems safe to assume this was a product of the rush to get the army out of camp and into formation), it is already too late for him to make any meaningful changes to his own formation. He is already committed to fight with the plan he has. That doesn’t mean, as we’ll see next time, that a general in this position couldn’t respond to sudden changes – Aemilius was at least aware that the Paeligni to his right were in trouble (Plut. Aem. 20.5-6), though he seems to gamble that they will hold; according to Plutarch he does respond to seeing the organization of the chalkaspides begin to break up by communicating to his officers that the component maniples of his legion should advance independently (so some giving ground, others pushing, to the degree they were successful, rather than presumably trying to keep one even and coherent line), aggravating the gaps into the Macedonian phalanx (Plut. Aem. 20.8-9).

As a result, while the general was certainly expected to think about and tweak the positioning of his forces, most ancient and medieval cultures had fairly standard battle arrays and formations which they essentially reused. It is by now more or less a commonplace to note, for instance, that Alexander’s dispositions at Granicus, Issus and Gaugamela essentially replay the same battle three times (refuse the left, phalanx anchors the center, Alexander leads the cavalry as the key striking force, breaking through on the enemy’s center-right rather than fully flanking them), exploiting the same weaknesses in the Persian armies that opposed him. The Battle of the Hydaspes is more complicated because of the difficult river crossing, but after that it the battle plan mostly another ‘Alexander-battle’ like the rest. The same is of course mostly true of hoplite battles or the Roman manipular legion: generals could and did innovate, but there was a ‘standard’ fairly ‘safe’ battle pattern for these armies and they more often stuck to it than not.

Instead the general’s role was if anything more complex and difficult: he had to manage an uncertain information environment, attempting to ascertain how his enemy intended to fight from incomplete and occasionally outright false reports and his own knowledge of how the enemy tended to fight battles. Approach marches and camp sites – ironically something Total War largely does not let you choose – mattered a lot more because they determined the terrain of the battle. But rarely did generals have the information necessary to make micro-adjustments to match known enemy dispositions (at least not before they might have become aware that a flank was crumbling, etc). Instead, they were forced to make decisions based on incomplete information and assumptions; in effect, they had to guess in advance.

Which is fine, because it was also fairly rare that commanders of pre-modern armies could exercise the kind of fine control of an army required to achieve perfect ‘match-ups’ even if they did know exactly where the enemy was. That isn’t to say the general necessarily had no role once the fight was started. Next time we’ll take a look at command: what is the general’s job in battle and how does it vary culture to culture?

  1. That is to say we are removing for the moment the strategic game including in many games unit production (or ‘macro’) as well as elements of games where a key aspect is simply feats of mouse and keyboard speed and dexterity (or ‘micro’) and focusing purely on the tactical elements here.
  2. Yes, I’m aware of groups with formation locking but again capable players do not simply formation-lock their entire army and move it into the enemy as a single unit.
  3. Important word
  4. Though I must call out the otherwise somewhat weaker depiction of the Battle of Philippi in HBO’s Rome for the wonderful moment when Antony essentially points out, to Octavian’s horror, that at this stage he has basically no control over the battle, so he can either sit and watch or charge in, as he likes. Though the opening giving of orders, as we’ll see next time is utter nonsense: two fellows shout and then someone lowers a flag no one can actually see and everyone understands that is the order to advance.
  5. I’m going to use this word a few times so it is worth defining: a stratagem (from Greek στρατήγημα, passing through Latin as strategema before arriving in English as ‘stratagem’) is a ruse or clever trick a general might use that is outside the more typical arts of generalship. This doesn’t mean that stratagems were always seen as dishonorable (on this note E. Wheeler, Stratagem and the Vocabulary of Military Trickery (1988)), but just that these were ‘special tricks’ one might play, usually involving disinformation or surprise.
  6. In part due to some recent bad weather limiting both armies’ ability to get a sense of their surroundings.
  7. A notable exception here were armies that were entirely mounted, even if they dismounted to fight. Such armies could, so long as their logistics held up, often effectively outrun the news of their coming. Such fully mounted armies larger than raiding parties are rare in agrarian societies, but common on the steppe, a crucial advantage to steppe nomads.
  8. Since setting camps on hills was often advisable, it is very common to have situations where both armies are encamped on different hills with the low ground contested between them.
  9. Chiefly Eumenes of Pergamum
  10. Put a pin in this, we’ll come back to it, but restraining victorious soldiers from rushing for the nearest loot was a perpetual problem for generals. Getting a wing that had broken through to turn on the enemy center rather than plunder their camp demanded discipline and command skill.
  11. Not the mythological figure, just named after him
  12. There is a tradition in our sources that this was a clever ruse by Aemilius; I think Johstono and Taylor are correct in rejecting this – it is clear from our sources that both armies had to scramble to respond to the battle once it started.
  13. Important word, given the unknowns here.
  14. Please note that hoplite spacing is much, much more speculative than Roman or Hellenistic spacing, which is also very uncertain so I am speaking here in broad generalities.
  15. That is, a unit which would be expected to maneuver independently. Many smaller organizational steps are purely administrative, while many larger ones are essentially operational in nature.
  16. And I suppose for those writing fantasy fiction, some magical means of visual amplification might well have the same effect.

232 thoughts on “Collections: Total Generalship: Commanding Pre-Modern Armies, Part I: Reports

  1. It seems like an author of fantasy-fiction could get a lot of mileage out of exploring the edge an otherwise realistic pre-modern army could get out of a few flying carpets (or pegasai or whatever), and a few wizards able to send messages long-distance via magic. Though you could add a fair amount of magic and still have commanders be constrained by the limits of the Mark I eyeball.

    1. When I designed the setting of my fantasy series I didn’t include “battle wizards”. No general in their right mind would waste such a valuable individual on the front lines, when they could be using crystal balls or magic mirrors to scry on the enemy, giving the general a perfect overview of the enemy’s makeup and position, then use magic messages to instantly relay their commands across the battlefield.

      1. Yes, but then the other side’s wizards are casting spells of masking and blinding, and filling the air with noise….Much the best use for a dragon is as an aerial scout (gets like World War I – where most of the contest in the air was about observation)

        1. Yes, actually! And then there are ways to get around that turn… Wizard “duels” are a complex game of baffling and scrying, spell and counterspell. It’s analogous to radio operators worrying about the enemy trying to crack their codes while trying to crack the enemy’s codes while the enemy tries to jam their communications while they’re trying to make their communications harder to jam.

          1. Or as Nakor in serpentwar puts it: “First wizard casts spell, enemy wizard counterspells, third wizard helps first wizard, enemy wizards join second wizard to strengthen counterspell, army comes around and cuts down allo these wizards standing around.”

          2. Magical death curses to target enemy wizards/leaders? Weather manipulation that can shift the balance of an entire battle, spells to manipulate luck?

            Magical CQC weapons to protect them from conventional assassinns?

      2. Spoiling as little as possible, there is a battle in The Wheel of Time where a general’s vision advantage, given to him by his magic users, allows him to carry out a battle plan that would be impossible without it.

        1. de Camp’s Clocks of Iraz (Unbeheaded King series) also has a siege turn on a scryer’s glimpse through a warded council of war.

      3. Depends on if battle mages are the equivalent of ww1 artillery, or even just early gunpowder artillery.

      4. A statement with a lot of baked-in assumptions about how magic works that don’t apply to all fantasy settings. In particular, while magicians are sometimes rare and valuable as water in the desert, sometimes they’re as rare and valuable as water at the beach. A setting where only one man in a thousand has the potential to use even the most basic sorcery will have a much different attitude towards battle mages than one where anyone can use basic battlefield firebolts with a halfway-decent staff and a week of practice.

        1. And on the power level of those wizards. In most editions of D&D, for instance, virtually no number of Level 1-3 soldiers would make a difference against even a handful of high-level spell casters. At that point, the military (and indeed society as a whole) would look vastly different from anything in real history.

          1. Honestly well I don’t think it would have as big of a impact let’s not ignore that in D&D Charles atlas superpowers are in play for the non spellcasters as well. For an example we can see things like rouges dodging giant explosions with uncanny dodge, barbarian having unarmed defense that let them go into battle and avoid damage from being just that tough or champion warriors who can regen hp when they have lost half of theirs. A good lord example of this is siege staffs meant to be the errebon artillery equivalent it does 2d8 damage very dangerous to most npcs in a army who might have 8-11 hp but very survivable to high level PCs

          2. Like I said, it does depend on edition. In the older ones, casters eventually hopelessly ouclassed non-caster PCs. But even if not, the point holds that the thinking about war would be vastly different in a world where 5 people can wipe out an army.

          3. All characters with levels are supposed to be rare.

            Nevermind that they can’t be, since the world is late modern in many significant respects and only magic can account for them. Women not spending all their childbearing years pregnant or nursing means there must be lower infant mortality than existed prior to the late 19th century at the earliest.

          4. How rare depends on the setting, but usually you can expect to find some level 2-4 people in any decent-sized settlement. If there’s a market worthy of the name there’s probably someone with Cure Light Wounds you can get in touch with. Remove disease is a bigger ask, though.

          5. Exactly, Mary!
            Almost nobody seems to think through the social changes needed for female warriors to be common as grass.
            Women weren’t kept in the home for the evulz. Reproduction and child rearing were vital to maintaining human population and civilization in pre-modern times. The fact that women didn’t get any status for this vital work was part of the cross cultural tendency to value male activities over female regardless of their objective value.

            And then there’s the physical disadvantage women have in melee combat, being generally smaller with less upper body strength. Sure there are women who can go toe to toe with a man physically but they are few and far between.
            There are forms of pre-modern warfare that women can excell at, mostly involving skirmishing rather than shock battle and ranged weapons. Historical women warriors tend to belong to horse riding, bow wielding cultures. There’s a reason for that.

          6. @Roxana: In fact the ubiquitous modern depiction of pre-modern women wearing pants ignores that there was an entirely practical reason why in most societies women wore skirts: that unless women can bath almost every day yeast infections would become a big problem if they tried to wear pants.

          7. More to the point, skirts do not have trouble expanding and contracting. That, even bathing won’t help with.

          8. @Roxana P.S. I’ve wondered about a fictional world in which some ancient Sumerian stumbled across the secret of gunpowder, and within a comparatively short time (~500 years) crude hand cannons evolved into muskets (made of bronze!) and the social changes that might happen given that a woman can shoot someone dead with a musket ball as easily as a man can. Maybe a sacred order of battle nuns or priestesses wielding firearms? IRL it happened with the Dahomey.

          9. Men had to achieve more to have children than women did.

            Even in Europe, on average, people have twice as many female ancestors as male ones. Therefore, it must have been harder to a man to marry at all, though those who succeeded found it easier to remarry.

          10. The fact that women didn’t get any status for this vital work was part of the cross cultural tendency to value male activities over female regardless of their objective value.

            This is an exaggeration; women could generally get quite a bit of status for having lots of children and being good wives, even if this didn’t compare with the status accorded to, say, a great lord or monarch. (Though of course, the vast majority of men didn’t get that kind of status, either — let’s not fall into the trap of assuming that the kind of people who are talked about most in the sources were normative for society as a whole.)

            In fact, the only period I can think of where “women’s work” arguably has *no* status is the modern (post-1960s) world, and this is the result, not of some universal cross-cultural tendency, but of a deliberate drive to get more women into paid employment.

          11. Fair point CJ. Wife and mother, mistress of a household was not a position entirely without status or other perks. In every society I’m familiar with a married woman is socially superior to an unmarried one, is economically better off and has some power if only over her own children and servants. But there wasn’t really anyplace to go from there. Your social mobility depended entirely on helping/pushing your husband to improve his. When both spouses shared similar ambitions that worked splendidly. When it didn’t things could get quite unpleasant for both.

          12. The vast majority of men had no social mobility.

            True; a society in which, due to technological constraints, 90%+ of the population need to work on the fields isn’t going to have many opportunities for rising up the social ladder.

          13. Depends on what you mean by social mobility. It was generally not possible for a peasant to become anything else but he could become a wealthier and locally influential peasant.
            The medieval English peasantry had a lot of mobility within their class. Serfs could accumulate land and gain status within their communities. Free men might over several generations rise to the gentry. Not easy but just possible.
            In ancient Egypt there were two paths upward for a peasant who made good. If one was lucky a turn as a soldier could get you loot enough to educate your son’s and education was the other path upward. Most scribes were low level white kilt workers but literacy was the path upward.

          14. >Women weren’t kept in the home for the evulz.

            Quite right. But there’s an ever more important point, that very very few people even realize.

            Pre-modern life was *hard*. As in, no mechanical aids, no communication beyond shouting distance, and brutally dangerous even at the best of times (ie when it was only nature trying to kill you).

            So I’d argue that women weren’t KEPT in the home, they were ALLOWED TO STAY in the home, while men were required and expected to do the hard jobs and take the big risks every day.

            Of course, child bearing would balance things out, and indoor work is still work – just safer and more comfortable. The point is that things are balanced out, women weren’t kept locked indoors while men enjoyed the air and sunshine.

          15. I read a story once in which a princess loved a stable boy who was the sole worker in a premodern royal stable. Even when many royal guests were expected, he didn’t hire help. Unreal.

          16. Warfare by Groups of Adventurers
            the “soldiers” are there to occupy ground and control the population

        2. about the characters with levels thing not being that rare im not sure how only magic can account for them? For example the three classes i listed all have subclasses that have nothing to do with magic. It is true that say wizards or artifcers who treat magic as a science would have benefited(as seen in errebon with dragonshards acting as a much simpler source of magic energies as one example)
          indeed the description for the champion subclass says “The archetypal Champion focuses on the development of raw physical power honed to deadly perfection. Those who model themselves on this archetype combine rigorous training with physical excellence to deal devastating blows.” as one example

          1. The post said most editions, and you are acting as if fifth is the only edition. In editions prior to 3rd (original game, early basic, AD&D1, BEMCI, AD&D2, 3.0, and 3.5 high level magic most definitely completely dominated over non-magical characters.

      5. I think it depends heavily on what magic in your particular setting is and isn’t good at.

        If using magic for clairvoyance is easy, but using magic to conjure up bolts of lightning is hard, wizards stay well behind the battlelines and use clairvoyance. If the reverse is true, wizards become the equivalent of Early Modern era field artillery.

        Magicians will see very different ‘doctrinal’ roles in a military depending on whether they are particularly gifted in, just off the top of my head:

        1) Clairvoyance, as discussed.
        2) Direct sending of magical messages, as alluded to.
        3) Telekinesis or raw elemental force manipulation.
        4) Turning into animals
        5) Summoning or controlling many relatively weak minions at a short distance.
        6) Summoning or controlling a few very strong minions at a great distance.
        7) Bolstering the courage or physical performance of one’s allies.
        8) Weakening the cohesion or performance of one’s enemies.

        Each of these is likely to result in a different ‘playbook’ of moves used by a combat magician to support (or substitute for) conventional armed forces.

      6. I’ve done a similar thing. Although you can use a wizard to make someone’s head explode, the unfortunate chap’s mate can then brain the wizard with a club and no more wizard. They’re much better laying subtle curses trying to bias the dice of fate against the enemy, or snatching glimpses of prophecy (accurate or not).

    2. It’s a reasonably common trope. It’s very convenient since it lets your heroes be outnumbered but win the day through clever tactics. It’s also good if the author needs the protagonists to win a lot of battles.

      1. And an enemy is going to try to find counters to this – maybe not effective ones at first, but they will iterate…

        1. Glen Cook’s Black Company goes to a fair bit of effort to hide the fact that they *have* mid-level wizards, e.g. faking spies as information sources to cover for scrying etc.

          1. It’s some time, that I read Black Company, but i remember them put a lot of afford in making their wizards look like bumbeling idiots, while they were in fact incredible powerful. Only seconed to some of the rather god like creatures they met along the way.

      2. The Belgariad prequels actually notes that the major wizard-character admits that he was no great general, but having access to flying scouts and telepathy gave him an incredible advantage when it came to coordinating forces.

        1. In The Wheel Of Time, magic and Seanchan fliers play a big role in maintaining an overview of the battle and relaying orders, to an increasing degree as time goes on. Though the battles also get more confusing when you’ve got people generating gateways attached at the battalion level.

          During The Last Battle, they figure out you can make a gateway parallel to the ground rather than perpendicular, so the general of the good guys sits in a tent watching the battle from overhead through a gateway on the floor.

        2. Wow, long time since I thought about the Belgariad but you’re right – this really is a major feature of how the sorcerors in Belgarath and Polgara’s novels interact with battles – working either alone or in teams to gain powerful information advantages (And control the information gaining of the enemy) far more often than directly interfering.

    3. Fantasy magic also has fun “firepower” considerations. If a wizard can engulf 20 square meters of front in fire you start having serious issues that did not plague pre-modern armies.

    4. I think it’s the opposite – the author could distinguish himself if he convincingly depicted a leader doing good scouting and logistics. In my opinion people are so used to the internet and smartphones they keep reinventing the same devices just to make communication easy and regular, like they’re used to. Magic mirrors, pigeons, crystals, telepathy, visions and so on.

      1. Donaldsons Mordaunte Series IIRC

        The Army marched without train, because every evening the camp equipment was send to them via mirror

  2. Put a pin in this, we’ll come back to it, but restraining victorious soldiers from rushing for the nearest loot was a perpetual problem for generals. Getting a wing that had broken through to turn on the enemy center rather than plunder their camp demanded discipline and command skill.

    I’d heard about cavalry in the English civil war doing this a lot, saw your comment on this battle (and remembered an earlier post), and wondered exactly this.

    Dominions series does something like what this post describes, you give a set of (up to 5(?) orders in the third game, not sure otherwise) to units and some general behavior, and an AI fights the battle.

    The total war checkerboard thing looks a lot like “game rules leading to unexpected consequences”, since it isn’t something I’d think of even being around computer games a good amount. I am surprised at how strong you are describng the rock paper scissors, since a game like this would seem to have other systems to create interactions/make an army stronger or weaker/let players handle well or poorly, though I’ve never played total war much.

    1. Dominions also does the limited intel thing. Without active scouting you don’t see troops more than a province away and you get a report of their numbers that’s somewhere between 50% and 200% of their actual value and you only get to know the most common troops in their army plus large or flying creatures. And that’s before taking into account stealthy troops or magic that adds/subtracts troops from the report.

      1. Dominions is actually a fairly interesting example (I mentioned it before) precisely because of the “set up the battle and watch it unfold without contro” combined with the “limited intelligence” (though sending in a scout, “pinging” lets you get the exact numbers, but remember that even those improbably accurate numbers will be *a turn out of date* and an enemy can react to your pings to potentially bring in more troops)

        1. Not true, actually!

          Only magical scrying and The Eyes Of God give you a perfectly accurate count. According to the wiki, regular scouts are +/-30% and spies are +/-10%. That’s not counting magical effects that actively distort your count.

          1. Oh, I just realized you meant actually initiating a battle with a solo scout. Yeah, that works, though the combination of time lag and simultaneous movement means it’s very likely the army in question will not be in the province in question on the turn you move your army there.

    2. One of my great disappointments with Total War is that marching into battle with reasonably realistic force compositions and dispositions is basically guaranteed to lose you a PvP match or even a campaign skirmish beyond a moderate level of difficulty. On the other hand, it’s no surprise that the series managed to achieve huge semi-mainstream success by stepping into a fantasy setting with the Warhammer games.

      A lot of the weirdness, IIRC, comes down to the way the morale mechanics work. Being flanked by enemies is a huge morale penalty while having friendly units behind you and to your sides is a strong benefit. On top of that, as Bret has previously discussed, ranged units tend to be much more effective at much longer ranges than in reality. So the big gaps between units exist to enable the rock-paper-scissors maneuvering Bret mentions here while simultaneously maximising the morale effects of unit adjacency and minimising the effects of enemy ranged fire and the enemy’s ability to inflict flanked penalties to the center of your line.

      1. Meanwhile I chug along with one solid line of melee troops, one solid line of ranged troops, and some cavalry running off into the far distance because I forgot to retask them after they shatterd their target.

        II, uh, don’t play competitively.

      2. It was pretty well accepted on, at least, the TW forums I used to frequent that the higher difficulties are only for those who enjoy min-maxing. For everyone else it’s either normal or hard. Not because it’s too difficult — just because spamming archers as your only tactic grows old fast.

    3. A lot of ancient, medieval and even early modern troops were in it for the loot. Generals understood that, they were after loot too. They just wanted their soldiers to finish the damn battle before they started looting!

      1. There’s also the opposite: Eumenes of Cardia lost to Antigonos when he won the battle but his troops lost their camp (and the accumulated treasure of 30 years of victory). The troops swapped Eumenes for the loot (Antigonos executed him, of course).

      2. I would imagine a number of generals at least tried to get this under control by telling everyone they’d split the loot up after the battle in some sort of process, so you don’t get extra if you start looting early and maybe even lose your share. Probably with limited success.

        I know the British navy eventually had rules for division of prize money, but I imagine it’s a lot harder to enforce a split when the loot in question mostly fits in pockets rather than being a ship.

        1. It evolved into an informal rule that troops had three days free rein in a city taken by storm. Also rules for fair shares and so on. But a lot ended up in pockets – merchants followed sepoy armies in India and swapped liquor or cash for jewels.

          Bear in mind that – repellently – most of the ‘loot’ in Greek, Roman or early medieval times was persons. Slave-buyers were always at hand.

          1. Repellent to us but up to say the late eighteenth century just business as usual. Women and children had been loot back to dim prehistory. The stratification and specialization of civilized states added men to the mix..

          2. Roxana – slave-taking for profit (on the Greek, Roman or Viking model) died out much earlier in western Europe (by no later than 1050). The Tatars pursued it in eastern Europe up to the mid 1700s, selling to the Ottomans; from my reading it was not a thing in India or China after 600 or so. Women and children suffered – but they were not led off en masse to the market.

      3. An extreme example of this was a unit called “Lisowczycy” (sometimes butchered to Lisowczyks in English). They were the irregular light cavalry of Polish-Lithuanian Commonwealth, noted for their effectiveness and cruelty. They just weren’t paid at all.

        They were typically sent after enemy supply lines, or to conduct the ‘small war’ in enemy territory. Not that it made much difference to them – they were known to plunder even friendly territory.

        1. The Ottoman bashi-bazouks were their counterpart, or the original hussars. I think you could find troops like this in every pre-modern army.

      4. I wonder if that situation contributed to the success of Chinggis Khan’s reformed forces. By collecting the loot centrally and redistributing it there’s less of an individual incentive to race off and grab some for yourself (because if you don’t, some other bugger might get there first!).

    4. Cavalry running off to plunder wasn’t a unique feature of the ECW, as it tended to happen any time soldiers saw an opportunity for loot (especially when getting close to the enemy baggage) unless those soldiers were highly disciplined. But the royalist cavalry had a particular reputation for it in the ECW and it may not be too much of an exaggeration to say that this tendency had a major impact on the outcome of the war: indiscipline of the cavalry had a decisive impact in at least two key battles while the war was still winnable for the king (Edgehill and Marston Moor).

      Somewhat unfairly, I think, this tendency is personified in the popular conception in Prince Rupert, but I would argue that he was a more careful and considered commander than his dashing reputation suggests. At Edgehill, for instance, the sources suggest he was well aware of the danger of leaving the flank exposed and made a serious effort to rally the cavalry following their rout of the Parliamentary horse (without success).

      Thereafter his advice to the king and his own actions appear to have been generally operationally sound (if not always popular with his peers) save for the decision to give battle at Marston Moor – admittedly a war-losing mistake, but and even that (the context of which is somewhat weird) was a battle that was there to be won had it not been for indiscipline of Byron and Goring’s commands (or if Newcastle had moved more decisively to assist Rupert in the prelude when called to do so).

      It’s also worth noting, of course, that prior to 1644 there is no reason to suppose the Parliamentary cavalry’s discipline was any better: the main difference is that they weren’t as good, so didn’t have so many opportunities to squander victories by chasing off after defeated foes. Rupert’s cavaliers do suffer in retrospect by comparison with the Ironsides, but they only emerged in 1644-5 and were genuinely exceptional.

      1. One reason why Sheridan managed to flip-flop a battle– his forces were routed, he returned, and he got them to rout the other side– was that enemy forces had broken up to plunder the camp. It was late in the war, they were hungry.

        It also helped that some had retreated in good order, and Early dismissed the warning that he had to drive them off.

    5. Battle orders getting out of sync is completely incidental – the design goal was to remove all interactivity from battles. Dominions is designed around simultaneous turns, which is a must for fluid multiplayer games. A->B->C->D style, sequential turns ruin multiplayer in turn-based games.

      A similar, but more granular and perhaps more interesting system is in the game Laser Squad Nemesis. A turn is 10 seconds, and you plan what each unit will do in these seconds, including *conditional orders*. For example “Go there for 3 seconds and fire grenade launcher at point X, but if you encounter an enemy retreat to point Y.”

      The concept of simultaneous turns that you plan ahead and play out in real time has also been used in Frozen Synapse and Frozen Cortex.

      ————

      Some limited information is given in RTS games like Total Annihilation (you can see units on radar but not what type of units they are) and Starcraft 2 (Terran building).

  3. which was part of by the Theban general Epaminondas’ decision to load up all of his elite Theban troops on the left wing at the Battle of Leuctra (371) was, in the Greek context, so innovative (this resulted in the Thebans crashing directly into the main Spartan contingent, which was Epaminondas’ goal).

    Typo – I think it should be ‘which was part of why the Theban general…’

  4. I didn’t see a link to Johstono and Taylor’s Pydna reconstruction.
    I am less interested in games (though fascinated by your evaluations of them) than I am in actual historical battles. I have often wondered about Zama. I’ve read that Hannibal advanced onto Zama and had to make camp where there was little or no water source, putting him at a disadvantage where he had to attack. Then his initial assault was with massed elephants. Surely by this time Hannibal knew that the Romans handled elephants by creating gaps in their line through which the elephants would naturally prefer to enter rather than charge into a mass of men. then the elephants could be individually dispatched by flanking troop detachments. Seems like the brilliant Hannibal was not up to his own standard that day– though as you point out, there was no doubt much that he did not know. Still to me, his battle plan seems wrong. I have to tell you I have never read Livy. I would be more than interested in your comments. One note: I love your posts. Unfortunately I am retired and living in Uganda and very broke. Hence I have not been able to contribute to your subscription. Sorry, I will rectify that if I ever recover my fortunes.

    1. Properly designed games are very good ways to explore what ifs to work at getting a better understanding of any given battle. The most accessible types of games, since you can actually see the designers assumptions in the rules. are either miniatures or boardgames. For the ancient period Dr. Phil Sabin’s “Lost Battles” or “Legion” or the 2 best at showing realistic tactical results. Dr. Sabin taught game design and ancient military history at Kings College London for many years until his recent retirement.

  5. An exceedingly interesting post. You’ve had me (1st) reading Johstono & Taylor and, when I thought I knew something, (2nd) checking and re-checking google maps street view. I have learned much less from whole history books than I do from a single post of yours.
    After being drawn by your lesson to spend so much time and energy, I will not resist making an observation (something appropriate to a pedant, I hope): Aemilius left the camp from a ridge, quite higher than the road. It’s not a mountain, much less an aerial view, but it must have given a somewhat better idea of what was happening. It’s a pity street view does not cover Kitros in more detail, although the hill can be seen from the same position you have posted, just looking back.
    You are doing extraordinary work, Dr. Devereaux. Thank you very much, and please, keep on.

  6. Instead of shouting orders, couldnt they use some kind of intrument? Just hoot a horn to signal the soldiers. like when to fall back and when to push.

    Also if they couldnt commicate with shouting, how did the romans cycle the hastati; the principes; and the triarii in the maniple system? Did the general just order the principes forward and then the hastati was told to fall back and let the principes take over?

    1. I’ll leave Brett to amplify, but Romans and medieval armies did use horns or trumpets, and nomad armies drums, to give simple signals (and Mongols drums and fire arrows and flags). But battle is really noisy, and there is dust, and most of the commanders are actually in the fighting.

      1. The Romans also used the cohort and legionary standards as a method of order conveyance.

    2. The immediate question this raises is: how many different orders do you want to be able to issue, how reliably will the troops be able to tell the orders apart (as the complexity of each signal increases along with the total number that need to be differentiated), and relatedly, how do you ensure that only the troops you want to follow a given order respond to it, if you’re delivering it via loud noise? If individual units within the army have different signals for when an order applies to them specifically, that further increases the complication. I’m sure you could have some basic signals, but I’d expect you’d want to have as few as possible, covering as general-purpose a set of orders as possible, in order to be able to quickly drill troops to remember them and then reliably recognize them in the heat of battle.

    3. Certainly they did use instruments. OGH is pointing out the ridiculousness of Hollywood signaling the entire battle with a single man shouting, and then a battle line a few miles across advancing all at lock-step.

    4. Also if they couldnt commicate with shouting, how did the romans cycle the hastati; the principes; and the triarii in the maniple system? Did the general just order the principes forward and then the hastati was told to fall back and let the principes take over?

      Each legion would have had its own commander, who’d have been in charge of that sort of thing. (Unless of course the general chose to personally manage a particularly sensitive part of the battle line, as Aemilius did at Pydna.)

    5. They do; it’s actually a very important part of military command and you’d have dedicated instrumentalists. Bret’s almost certainly going to cover this in much more detail than I know, but the main problem is that you can’t really get much more complicated than “advance” “retreat” and maybe “Principes forward, Hastaii back”. There’s no way you’re getting even an army that does drills to memorize and reliably interpret in the heat of battle a command like “Third cohort pull back four hundred feet, second cohort advance and push right, first cohort hold position, cavalry stop raiding their camp and get back here!

      1. Having one of your few order signals be “Cavalry Get BACK HERE Right Now!” might be useful — except for that whole ‘never give an order you know won’t be obeyed’ thing.

  7. I can’t really recommend it because the game is glitchy and has limited replay value, but there’s a very old game “Riders of Rohan” (Can see/get it here http://www.abandonia.com/en/games/211/Riders+of+Rohan.html) Which does a pretty good job of limiting both information and command and control over a battlefield, despite the top down camera view. One of the reasons I still remember it despite being fairly lackluster in a lot of regards.

  8. “Because forming up for battle takes a while, it was generally impossible to wait to see what the enemy’s formation was before adopting your own… So the basic question of the overall formation has to be made effectively blind to the enemy’s formation.”

    Sort of like football, really: neither huddle knows what formation or play the other side is setting up; they just have to use educated guesswork. Except that an ancient general’s “playbook” was rather more limited.

    1. In the board game Stratego, you set up your pieces without knowing how your opponent is setting up theirs. (The pieces are only marked on one side so you can identify your own pieces but not your opponent’s.)

      In the board game Cyvasse, you also set up your pieces without knowing how your opponent is setting up theirs. (There’s a screen in the middle during set-up.) This game started as a fictional game in Song of Ice and Fire, but somebody made a real version.

    2. Actually, in American football both sides sometimes change formation before the snap, and evaluating the response to the changes is an important part of play, but there is a limited amount of time allowed before the ball is snapped so making complicated formation changes risks penalties and confusing yourself more than the opponent.

  9. In Starcraft, scouting gives you partial-but-correct information, but the correct information can still be a ruse. Like other strategy games, Starcraft heavily relies on unit counters, so a key aspect of the game is to know what tech path the enemy is using. So if I play terran vs. zerg and see a spire, it means zerg is producing mutalisks and I need to prepare good anti-air defenses in my base – mutas are a fast hit-and-run unit, ideal for raiding parties. But wait – the spire is going to be well-hidden. So maybe instead of scouting the spire, I scout how many gas geysers zerg has; Starcraft has two resources, minerals and gas, and the rule is that higher-tech units cost more gas, so if I see zerg is mining six geysers then I can expect mutas even without seeing a spire, whereas if I see only four or five then I can expect something else, like hydralisks.

    Now, this is completely different from actual historic warfare. Tech changes like this don’t really exist; even in modern warfare, they matter long before battle. But the part about dealing with extremely sparse information and having to fill in details based on what you know of strategy is there, it’s just that instead of historic strategy the strategy you need to know is the current meta. In a way it’s not too different from what Epaminondas did in Leuctra – Epaminondas relied on the hoplite meta in which the right has more prestige than the left and hacked it to defeat the Spartans, and much of high-level esports likewise relies on knowing that your opponent is a skilled player who understands the meta and using that to your advantage. (And if you’re facing a noob – defined here as 99% of the player base – then you use that knowledge to your advantage, just as Roman commanders used different strategies to deal with silly colonial uprisings from those they’d use to deal with real enemy armies.)

  10. It is a great rule in fiction to never depicting the planning AND carrying it out, except to dramatize how wildly bad the plan goes. Otherwise it’s repetitive and so not dramatic.

    1. You could imagine a sort of “Oceans 11-esque” battle sequence though where one side constantly appears to be making the right moves, and then everytime you see that you get a brief flash-back to its counsel meeting the night before where the hero-protagonist says “I expect the Imperial Army to deploy their heavy guardsman here so deploy the ..” . Although I am sure that would get old fast.

      1. It indeed sounds fun, as a gimmick for one part of a movie or a single episode of a show, as part of setting up a character as a good commander, but not great to use long term.

    2. I’d expect that looking through the MICE system, the council of war is more interesting than the battle for C stories, and the two are comparable in M and I stories. Especially if you make the organization inhomogeneous (e.g. transplanting the modern staff system, assigning intel, signals, logistics, etc. to a stripy tribune each), — or perhaps a bit dysfunctional, the various characters not trusting each other due to actual and/or perceived incompatibility of goals and thus withholding/dismissing information and having to negotiate cooperation with the plan. Probably only a very cynical E story could pull off “let’s pause the action until all the obstructive people are dealt with in one fashion or another”, unless of course the main plot is about fixing an organization.(*)
      MICE :
      – Milieu (from protagonist arriving to protagonist leaving or choosing to stay);
      – Information (from an open question becoming known to finding the answer — except by tradition, the detective genre has a lengthy introduction section before the dead body is found);
      – Character (from protagonist deciding to change their social role to success or acceptance of failure);
      – Event (from protagonist deciding to try “fixing a thing that is wrong with the world” to success or acceptance of failure).

      *: Has anyone written / would anyone write a story about “FDA delenda est”?

        1. A lot depends on what you’re trying to show. So, for instance, Barney Miller, an 80s cop show, repeatedly dealt with the aftermath of shootings, but I don’t believe it ever actually showed a gun being fired, because it wasn’t interested in the action scene. It was interested in the effect the action scene had on the characters.

          So we never see Chano kill two bank robbers, we just see the near breakdown he has afterward.

        2. Yes, that’s exactly what I’m saying. If the primary tension(s) in the plot is/are between characters who end up fighting on the same side, then the barbarians at the gate can be merely a plot device forcing the characters to temporarily cooperate (or betray each other in hope of gaining some reward), and the actual battle can be omitted in favor of the planning. (If you’ve ever been to a general meeting of a condo (or homeowners’ association) and, between facepalming, had the thought “I should have brought popcorn”, that’s the idea, just with increased meaningfulness because it’s a matter of life and death, glory and ignominy, rather than a petty squabble over fees.)

          For that matter, if the story is largely about showing off “the scenery”, the worldbuilding, the culture(s), the council has enough story-relevant stuff to make the option of showing it rather than the battle appealing. Especially since it happens at a pace that an actual human observer present would be able to follow, unlike a battle, thus lending itself to writing more easily.

      1. One good example is probably the Horus Heresy/Siege Of Terra and War Of The Beast series from the WH40k franchises. The stories overall would fall into E, I guess, but the war council scenes are largely about character interactions and development. Horus’s councils show him starting as a charismatic paragon who is beloved by all albiet a bit full of himself, falling further into pride and demanding synchophancy and effectively gutting the independence of the group of four people who used to both serve as the advisors who’d (privately) really talk back to them and say things that need to be said but he doesn’t want to say himself like harsh criticism of an officer who isn’t being demoted. It also lets him showcase his belief that he’s in charge, not the gods, not his brothers, and certainly not any priests. Then by the time of the Siege he’s largely deyached from the material aspects of the war (he’s busy with the warp-related aspects of the campaign) and the command structure of the traitor armies has fallen apart.

        The War Of The Beast scenes are basically Vangorich growing increasingly fed up with the High Lords’ incompetence and internal power plays while working to focus them on the fact that the Orks have a moon coated in guns in orbit over Terra, until eventually he decides to just murder them all.

        In both cases, the scenes elide most of the actual strategizing except in the most general terms; most battle plans are revealed in narration as they happen. I think the most detailed is the vote on the founding of the Deathwatch, where Koorland tells the High Lords why he wants a force of small, mixed chapter strike teams and they’re going to refuse because they see it as a political power play until the Orks suddenly reactivate their Attack Moon in orbit and they grudingly vote yes while Vangorich adds to his list of reasons why they should die.

    3. And of course, almost all stories will prioritize depicting the carrying-out over the planning, even if the plan goes perfectly. Even in non-visual media, planning is less exciting and less tense.

      I wish there was a good solution to this.

      1. Of course LotR had the whole Council of Elrond as exposition: of who the major players were; what intelligence they could bring to the board; considering and rejecting strategies; and finally coming up with a battle plan. Which barely got out of the starting gate before going down the toilet. All that survived contact with the enemy was “somehow, get the Ring Bearer to Mount Doom by stealth”.

        1. Of course, Sauron’s scouting-and-recon was even worse; the last fix he ever got on the Ring was when the Company escaped from Moria and disappeared into Lorien; and aside from sending a company out which set up a failed ambush at Sarn Gebir, the Dark Lord spent the rest of his existence flailing blindly at false reports and bad guesses.

          1. Which Aragorn exploits after the Battle of the Pellanor Fields, with the play of taking every last soldier that could be scraped together towards the Black Gates to deliberately mislead Sauron’s attention; Sauron knew who Aragorn was (thanks to the Palantír), and couldn’t imagine the One Ring *not* being somewhere among Middle Earth’s Mightiest Heroes.

      2. You can intercut the planning with carrying out each step, or do it in voiceover or narration. Very common in heist movies.

        Come to think of it, A New Hope and Return Of The Jedi devote a limited but nontrival amount of screentime to their basic battle planning.

      3. ESPECIALLY if the plan goes perfectly. That is the dullest, because it’s all repeating yourself. Order of the Stick has many battle plans that go awry, but the one where the Order does it perfectly opens from the villains’ point of view because it comes as a surprise to them.

  11. Fantasy wargame Dominions (5 being the latest iteration, obsoleting the previous versions) features many of these sort of considerations. The mechanic that does a lot of the heavy lifting is simultaneous turns: players can choose the deployment of their forces and give each squad and commander a set of scripted orders, but battles only take place during turn calculations once each player has submitted their moves and there’s no direct control, players instead receiving a report and a battle replay on their next turn. Consequently, while it doesn’t capture nuances like offering battle or choice of terrain, it does mean battle plans have to rely on guessing what the enemy is going to do (or having seen the army deployment previously and guessing the opponent didn’t change anything), and a key challenge is coming up with robust plans that will work out even if the enemy does something slightly unexpected (like deploying their more elite units on flank different from the previous battle), and accounting for the possibility your troops will not be able to execute their orders and going off-script.

    Scouting information is another key point. If you do engage in a fight, the information contained in battle report is exact (this opens up possible strategic considerations like invaders deliberately masking their exact force composition and deployment by capturing provinces with skirmishing forces before the main body moves in, and defenders trying to force a battle with expendable forces to get said report), but if you don’t have such luxury, you have to rely on scouting reports that aren’t just incomplete but may contain errors, although accuracy is increased if you have scouts deployed in the province, if the province is loyal to you, or if you use various magical means of scrying. Moreover, due to the simultaneous turns nature of the game, even if you have scouted out the exact composition of a field army, the enemy could still be deploying troops with high strategic mobility (including magical means of transporting entire armies) or stealthy movement to reinforce, so you still can’t be quite sure what you’ll really end up facing should a battle occur.

  12. The ‘reports’ section reminded me of the (somewhat generically named) ‘Waterloo’, by PSS for the Atari ST (Amiga/DOS).

    You act as Napoleon, receiving reports and issuing orders.
    The orders are sent via man-on-horse mail, and may well not arrive. Even if they do, they can be misinterpreted or ignored.
    It does have a (rudimentary) 3D view of the battlefield, but it is fixed on what you can see from your current HQ.

    Much to my surprise, you can even still get it on Steam:
    https://store.steampowered.com/app/1157860/Waterloo/
    The screenshots there show the kind of information you are working with.

    And there’s even a playthrough here!
    https://www.rockpapershotgun.com/heavily-engaged-waterloo

    You type your orders out in natural language rather than choose options from a menu, and the ‘misinterpretation’ aspect turns a flaky parser from a bug to a feature.

  13. I’ve tried the “realistic general” option in Total War games every now and then, but it’s just a bit pointless, isn’t it. Not only do you still get a much better view of the battlefield than an actual general would, as you said – the idea just doesn’t work when none of your units will do anything without a direct order. Maybe this would work better in a different game with a robust officer AI where your units could at the very least turn towards a charging enemy, because as things stand now you don’t merely play as the general in TW games, you play as a hive mind encapsulating every single decision maker in your army. I’d like to see a game properly tackling pre-modern generalship, though – I think it’d be fun.

    1. There’s a game called (I think) Grand Strategy: Civil War, which has an interesting system whereby units take longer to react to orders the further away they are from your general, to simulate the time taken for your messengers to gallop across. Though IMHO it’s somewhat flawed due to the fact that you’re still activing as a TW-style hive mind, meaning that your units will act in illogical and suicidal ways without your being able to stop them (e.g., if you order your cavalry to reconnoitre the other side of a hill but forget to uncheck the “Stick to roads wherever possible” box, your cavalry could well end up marching along a road between the two firing lines and getting shot to pieces in the crossfire instead of taking the more direct and safer route behind your own line, and you can’t reroute them in time because they’re too far from your commander).

  14. As is often the case, this gets me thinking about tabletop wargames design. I wonder if you could simulate the pre-battle phase with a sort of bidding system where the players would be presented with various combinations of battlefield conditions, deployments, terrain placement, etc. Presumably the more onerous the conditions you accept, the more prestige you would gain in the event of a victory. “OK, I will start with a numerical disadvantage, but I get to deploy on this conveniently located hill.”

    1. There are several rules for miniature wargaming with this kind of rules. Another interesting option is (relatively) random composition of the armies. Those efforts will leave you very far from what the post explains, though

    2. It’s sci-fi, and oddly enough probably the sort of situation that this sort of setup wouldn’t apply in (I imagine those ships can communicate with each other pretty quickly if needed) but you might want to check out Gratiuitous Space Battles. The entire game is about the way you want to build your fleet and a simple set of orders you can give them before the two fleets slam into each other, although there isn’t much in the way of terrain or different battlefield conditions. But you do have a blind sort of setup, which in some ways very well mimicks a pre-modern commander’s job.

      1. Yes and no. I wanted to mention that game myself — indeed the player can only give orders and select the initial placement of units, then watch as the battle unfolds — but for each scenario, on the lower two (of three) difficulty levels some information about the enemy order of battle (namely, each hull’s type and its position — but not its equipment or orders) is displayed while the player sets up. Separately, ship design and unlocking parts (incl. hull types) for that is a major part of the first few levels. (The equipment in the game is not balanced very well, thus only a few options are competitive. After the player unlocks those and designs the ships to use them, there is no particular gameplay reason not to keep those for all unconstrained scenarios, and sweeping clean a few scenarios gives more than enough reward to unlock everything else, including all hulls for all factions. For that matter, the factions are approximately balanced thus there is no reason to switch, unless the player likes the ship design part of the game, or wants the battles to look confusing, with both player and enemy ships being of the same faction — AFAIK no scenario features enemies of the player starting faction.)

    3. Various existing tabletop wargames approach some of these concepts (but usually only a few). In De Bellis Antiquitatis, for example, the sequential deployment is probably unrealistic per the points in this article, but the combat dicing system does something toward the idea that the genera doesn’t know how his own troops are doing. Any given unit could break without warning on the next roll, and we would retroactively describe them as having been in bad shape. Something like the Armati system (popular around 1995) has simultaneous deployment behind a screen across the table, so that is more “realistic”, and maneuvers are limited, but you do know how much fight is left in each unit. The even older WRG 7th edition ancients rules had a comparison of scouting potential to influence deployment, etc…

      1. Heck, while Warhammer (the tabletop game) has tended to streamline towards the more gamey (and balanced) formats, at least as far back as 5th ed. (and much more extensively in the actual Oldhammer days) there were entire slews of alternate deployment rules, including various variants of blind deployment

    4. A number of board wargames invoke aspects of this reality in some part of their system, usually ones of the “block” subtype. For example, Rachel Simmons’s “The Guns of Gettysburg” gives players the ability to see only that the enemy has troops in a particular space on the map and roughly how many, and it’s only when they contact each other through fighting that the labels on the little blocks are shown, and your reinforcements are randomized, and so on. Matthew Calkins’s “Sekigahara: the Unification of Japan” also gives you unpredictable troop loyalty for its operational-level scope, etc.. Obviously, the limitations of the form of the board itself shape the extent to which this is implemented.

      Miniatures games tend to do much less with this in my experience, in part because the point of the game is to have the miniatures on the table.

    5. You can get a lot of deployment uncertainty in an ancients miniatures game, by having multiple players per side. Then couple that with a staggered semi blind set-up. You really learn a lot about potential C2 problems. Part of the reason I don’t play computer games.

    6. I think “Lion Rampart” has a similar rule. You can make “oaths” before the battle (bets about certain things happening) that influence the amount of victory points you get.

  15. This isn’t just a thing for pre-modern armies, either. Generals in the American Civil War, my main area of study, had a devil of a time just figuring out where troops where. Armies could be strung out for miles on badly-mapped terrain with unmarked roads. Marches frequently happened in the dark, and consequently *nothing* ever happened on-time.

    Referring back to the issue of the WW1 analysis from Our Illustrious Blog Author Dr. D., I some often felt the primary problem of trench warfare wasn’t firepower or manpower but the impossibility of maintaining communication and cohesion after achieving a breakthrough. Also that the armies probably couldn’t organize the troops in a good way to achieve the breakthrough and preserve their organization anyway. The Bite-And-Hold maneuvers absolutely short-circuited the retaliation response, but they also involved, smaller, more tightly-organized attacks that controlled the chaos and kept in contact with the parent army, preserving communication.

    For all that military nerds love exploring the interesting world of firepower and armor in WW2, I think most historians argue that the key innovations were things like the truck and the radio. These are obviously used for logistics and communications. A tank can make a breakthrough possible or exploit one already made, but it will not hold that breakthrough without support. It also allows units to coordinate over long distances and to unite strategic and tactical choices in a way never previously-possible. For example, in WW2 it was possible to take an infantryman’s report to an officer a thousand miles away, then use that information to dispatch an air attack to support another formation entirely, within hours. That’s simply not something you could do in WW1, let alone earlier.

    1. The American Civil War has an additional fun layer of the fog of war – how do you know which one of your *generals* is any good, if you’re Lincoln or Davis? Generals would deflect blame to other officers or to uncontrollable conditions like weather / lack of supplies, and attempt to steal credit for successes. An officer saying another officer failed might be exposing a weak link, or might be pursuing a petty vendetta. Obviously you can just wait a long enough time until deciding that McClellan has had enough chances, but that hurts if they really were incompetent. It took Lincoln awhile to find Grant. (And Davis erred in distrusting Johnston.)

      I imagine that isn’t as big a deal in the ancient world where generals are also political leaders and civilian control of the military is weaker, since nobody is going to fire themselves. (I guess there’s also the fog of war on which units are actually good – if your Greek allies say they’re sending you their elite best-of-the-best, did they really? Hard to say.)

      1. Some good point. I’d have to consider it.

        I do see similar issues popping up even in ancient sources, both positively and negatively. For example, Varro seemed like the Man of the Hour. (Unfortunately his hour was Cannae.) Then again, a tired Senate threw the young Scipio a command in Spain that seemed like a forlorn hope, but he then turned the war around and effectively wiped Carthage off the map. I think Medieval sources also record internal sniping and difficulty finding capable subordinates, too.

      2. I imagine that isn’t as big a deal in the ancient world where generals are also political leaders and civilian control of the military is weaker, since nobody is going to fire themselves.

        Depends on which state you’re talking about — the Roman Empire, for example, sometimes went several generations without an emperor personally taking the field. And of course, even a great warrior king would need subordinates to keep things safe in other parts of his realm while he was out campaigning.

        1. I think all the emperors from Augustus to Marcus Aurelius were present at campaigns – and often battles. Given the short-lived Roman ‘dynasties) rarely more than 2-3 generations) and that most came to the throne through civil war, not sure where the ‘several generations’ comes from. After all, Romes was a soldier-republic and then a very martial empire.

          1. I was thinking mostly about the late Roman/early Byzantine period, when (IIRC) no sitting Emperor between Theodosius and Maurice led a campaign.

            Even during the early period, though, it wasn’t actually that common for Emperors to lead an army in the field:

            – Augustus: Fought most of his campaigns before becoming Emperor. After becoming Emperor, he led an expedition into Spain in the 20s BC, but I don’t think he commanded in any wars after this.
            – Tiberius: Had a long and distinguished military history, but didn’t lead any campaigns after becoming Emperor.
            – Caligula: Campaigns against the Germans and Neptune, which were widely regarded as a bit of a joke.
            – Claudius: Briefly commanded in the British campaign, before hurrying back to Rome for fear that people would start to plot in his absence.
            – Nero: Didn’t lead any campaigns.
            – Galba: Murdered a few months after becoming Emperor, didn’t lead any campaigns.
            – Otho: Led an army against Vitellius, defeated and committed suicide.
            – Vitellius: Defeated Otho, then defeated and killed.
            – Vespasian: Was in command of the Jewish War when he became Emperor, but then went to Rome to take up ruling the Empire, didn’t fight any campaigns after ascending the throne.
            – Titus: Left in charge of the Jewish War by his father, but had come back to Rome before Vespasian’s death. Didn’t lead any armies as Emperor.
            – Domitian: Led a single campaign against the Germans.
            – Nerva: No campaigns.
            – Trajan: Lots of campaigns in Dacia and the East; the most martial Emperor of the early imperial period.
            – Hadrian: Led a campaign against the Bar Kokhba Revolt. Didn’t command any other wars in person, but his extensive travels would have helped him get a sense of how well his governors and generals were doing.
            – Antoninus Pius: Didn’t leave Italy at any point in his 23-year-long reign, no military campaigns.

            So even during this period, whilst Emperors did sometimes go on campaign, this was by no means a normal occurrence. Even those who did lead armies usually only did so once or twice, to win a bit of glory, entrusting the rest of their wars to subordinates. With the exception of Trajan, and possibly Hadrian, I don’t think any of the early Emperors commanded or travelled enough to get round the fog of war aspect of knowing whether a subordinate’s poor performance was due to incompetence, bad luck, or unusual enemy competence.

      3. Battles are rare unless you’re in the army of someone like Alexander or Genghis. Pre-industrial armies could not afford to run large scale training exercises. So ancient and medieval states frequently didn’t know which generals were any good either.

        It might have taken Lincoln a while to fire Grant, but aristocratic states in ancient and medieval times weren’t a meritocracy and couldn’t fire generals for incompetence at all.

        Eg the 1415 CE battle of Agincourt where the French Marshal Boucicaut and Constable d’Albret, experienced soldiers, drew up a plan (we have copies today!) whereby they would stand on the defensive and force the English, who had run out of supplies, to attack them. They were over ridden by the completely inexperienced but higher ranking French royal princes who joined the army.

      4. There’s also that medieval people really did not like being ordered about by anyone but their own local leaders – and the leaders were touchy about rank and precedence. The ability to persuade, cajole or over-awe was as important as tactical prowess – often more important. The Romans were unusual even in the ancient world for their discipline. Interestingly this has echoes in the modern world – one of Eisenhower’s great strengths was his ability to manage a coalition force (another was his insistence that his generals keep in mind that they commanded a citizen army). By contrast, German generals were generally woefully bad at getting willing cooperation from their allies.

    2. The Union army had a balloon corps. Its job was to float barges up and down the Potamac and send up men in hot air balloons with binoculars.

      Sure, the Confederates figured it out quick, but it was so much of an advantage to know the enemy could not be on any route in sight of the balloon.

      1. And the telegraph. Lincoln did not have to wait weeks to hear the outcome of a battle.

        1. The telegraph being fixed IIRC had some fascinating implications, because some batltes would be reported immediately, but stuff that’s just slightly off the wire could still lag considerably.

  16. AD&D 2nd edition had spells like ‘Hallucinatory Terrain’, ‘Mirage Arcana’, and ‘Screen’, aimed at making areas look like they weren’t (including maybe to other wizards with crystal balls…)

  17. I feel like King of Dragon Pass might actually have one of the better representations of pre-modern generalship in current media.

    You know the size and composition of enemy forces, you can pick a very general tactic and goal, and sometimes you get access to unique opportunities, but you can’t really control the battle beyond that, and what forces you already brought.

    Of course, it’s a fantasy setting, so there are also some magical elements, but it’s pretty grounded in what you can do most of the time, with magic just being an additive element to your strength, albeit a pretty powerful one.

    1. Don’t forget the pre-battle sacrifices, which can be quite important. (As well as the option of doing an immediate charge with the hope of disrupting enemy sacrifices). Granted, the fantasy element means that Orlanth and Humakt really do watch and favor you if you cut up a cow before the battle, but pre-battle religious ceremonies were enormously common and well attested.

      1. Though I imagine thinking Ares was going to favor you for cutting up a cow before battle was important for morale.

        1. That’s one of my favorite quotes from this blog: “An American general who slaughtered a goat in front of his men would not reassure them, a Greek general who failed to do so would terrify them”

          1. I don’t know about the first part of that. If I saw Mark Milley hold up the still-bleeding head of a goat in one hand, a gore-covered knife in the other, and tell me to go and slaughter everything in my path you can bet your last boot I’ll be be *very* motivated to attack.

            Although granted, it won’t be for the exact same reasons as the ancient soldiers.

      2. Very true! Of course, one could argue that at least some of that effect could be seen in a materialist history as well, simply as a function of morale from being in right relationship with one’s deities.

    2. I was thinking about that game as well.
      I think the “Ring”-mechanic, of that game could make a fun version of the war-counsil, as it is described in this post. The player has to draw up a battle plan, and has to check in with his subordinates, to a) make the plan clear, and b) get everyone on board.
      You could have stuff like an very hot headed leader, who need to be convinced that he needs to be in the rear (and may still attack early), and some timid militia headmen who needs to be convinced that he needs hold the center for at least some hours.

  18. In defense of Henry V, the play is about Henry, not about Agincourt. And one bit that it gets right is how after the battle Montjoy shows up and Henry has to ask him who won. IIRC, the historical Montjoy and his English counterpart watched the battle from a hill, giving them a much better sense of what had happened.

  19. As a nitpick, spacing tends to be much wider in the warhammer total war games than the historical because of the presence of powerful magic that can severely penalize having too many units too close together.

  20. Just one minor thing, while most of it still holds up across Total War in general, Warhammer *is* an early-modern (at least the early bits of early modern) era game. The exact timeline and relations to history gets wobbly becuas eof rule of cool, but generals in Warhammer *do* have accesst o inventions (even without bringing in magic and flying creatures, a Warhammer general can *very well* observe the battlefield from a literal flying horse or griffin after all) like spyglasses. (I also note that Warhammer battles are actually on the slightly smaller side for TW games, for various reasons, though the points about size still stands.

  21. Fundamentally the tactical portion of Total War games (and indeed many real time strategy – RTS – games) is oriented around the concept of the ‘match-up.‘

    Alternative citation: https://tvtropes.org/pmwiki/pmwiki.php/Main/TacticalRockPaperScissors

    One exercise I do with my warfare classes is to form up the front row of students to make up a single half-file of a Macedonian sarisa-phalanx; without fail one needs to budget several minutes for this fairly basic task of “form a line one arm’s length apart facing this way.”

    Man, I wish I had Prof. Devereaux for a history teacher.

  22. Not to contest the legitimacy of anything that’s said here but the chequerboard formation (rightly) criticised in the opening, being a feature of high-level play, is as much as anything a response to the meta, and therefore represents probably the most exaggerated manifestation of the differences between TW-style battles and real life ones.

    Against the AI on lower difficulties you can use approximations of historical tactics and formations with great success, and that’s what I tend to do (albeit there is eventually almost always some element of unreality in manoeuvres over the course of a battle given the “deathmatch” nature of TW battles, where combat often continues long after real armies would have withdrawn).

    I play the games for enjoyment and that enjoyment principally derives from smashing armies together in ways which at least vaguely approximate the way that they might have done historically (or, in the case of Warhammer, the way they might do on the tabletop, which is generally a much smaller battlespace than in the video game and therefore mandates at least a little more in the way of realistic manoeuvring). Micromanaging units in a totally unrealistic formation isn’t why I play, I don’t find it fun, so I don’t do it, and it works fine, provided I keep the AI on a leash.

    Of course, this is something that is really only available against the AI (or other players with a gentleman’s agreement) and if you’re seeking to challenge yourself as a gamer then you will inevitably seek out the highest game modes and adopt unrealistic tactics accordingly. To the extent that these games impact popular perception of battles, too, the prevalence of chequerboard nonsense is still a problem, and I gather that is a large part of the point in articles like this.

    But it is still something which at an individual level at least is reasonably easy to fix, as you can maintain an element of challenge to the gameplay by placing voluntary restrictions on yourself.

    At a game level, i.e. something which stamps out this stuff for good, it’s a bit harder, because a lot of the things that make this difficult are baked into the game in a manner that you wouldn’t want to remove, and others in a way which are difficult to replicate, but three options that immediately come to mind are (1) a return to mandated choice from a list of pre-set formations as used in Shogun 1, rather than allowing complete freeform deployment; (2) serious morale penalties for units that venture too far from the nearest friendly unit; (3) make the battle map smaller.

    On that last point specifically, given that battles only take place where one side has managed to force it, it is in a sense ridiculous that the map allows the party that doesn’t want to be there enough space to run around for ages trying to avoid fighting – that is surely represented at the campaign map level. Part of the this may be down to the scale problem that afflicts TW: armies, for largely practical reasons, are not represented on the battlefield on a 1:1 basis at “real” size (your computer couldn’t handle it), but for largely aesthetic reasons, the battlefield is represented as though they were, so armies are much nimbler relative to terrain features like trees and buildings than they really should be. That probably can’t be fixed in itself short of a return to sprites (nah) but it can be mitigated.

    Whether the playerbase at large would actually respond favourably to any such attempts is of course a different question.

    1. I don’t want to say you’re wrong, but you’re missing the point, and I think you summarize why pretty well yourself.

      But it is still something which at an individual level at least is reasonably easy to fix, as you can maintain an element of challenge to the gameplay by placing voluntary restrictions on yourself.

      See, the problems Devereaux is pointing at aren’t about individuals. They’re about how media impacts the general public’s understanding of history.
      Yes, someone who knows how armies historically moved around can make armies move around historically. But that presupposes that someone knows how historical armies worked. Someone who doesn’t, obviously can’t. And more significantly, the unrealistic formations will influence how such people look at history.

      Strategy games allow people to experience what it’s like to be a general. If the sorts of things a player does and can’t do reflects in some way what real generals had/have to do, it can be a good educational experience! But if it’s not, it can give people the wrong idea about what war was/is like, especially if the ways that it’s wrong lines up with ways that other games, movies, etc are wrong (and areas which history classes don’t cover because they’re harder to test on than dates and names).

      Media does a great deal to shape how we see the world, especially the parts we don’t see with our own two eyes. The fact that people have access to resources that can set them straight doesn’t change this if those resources aren’t going to be used by a significant portion of the population.

      1. I do think it’s a bit unfair to use Warhammer 2 as an example, for a variety of reasons: The command problems (or lack thereof) are the same in eg. Rome 2, but the meta in terms of formation, etc. was fairly different (for a variety of reasons)

        1. Came to post the same thing. I haven’t played and of the WH:TW games, but from what I’ve seen of them, they seem quite a bit more “gamey” than the historical games, especially the older ones. Putting eg your Shogun 2 Yari Ashigaru in a checkerboard formation seems to me a surefire way to lose the battle instantly due to mass rout (because of uncovered flanks).

          A minor nitpick obviously, as the article is still solid – in fact, I’d rate “too much control and information” as a number 1 issue that undermines the “realism” of TW games – but there…

        2. So Devereaux should have ignored the most recent entries in the series in favor of one which doesn’t as clearly illustrate the problems running through not just the series but the genre?

          1. Yes, because Rome 2 *does* more clearly demonstrates the problems, because it isn’t messed up with other, warhammer-specific problems.

  23. Thanks for this interesting new collection! A couple of typos:

    “Armies had the move have to forage for supplies…” == Armies ON the move

    “I particularly light how the dust is handled..” == I particularly LIKE how

  24. The ‘reports’ section reminded me of the (somewhat generically named) ‘Waterloo’, by PSS for the Atari ST (Amiga/DOS).

    You act as Napoleon, receiving reports and issuing orders.
    The orders are sent via man-on-horse mail, and may well not arrive. Even if they do, they can be misinterpreted or ignored.
    It does have a (rudimentary) 3D view of the battlefield, but it is fixed on what you can see from your current HQ.

    Much to my surprise, you can even still get it on Steam!
    The screenshots there show the kind of information you are working with.
    And there’s even a playthrough on Rock Paper Shotgun (see Heavily Engaged: Waterloo)

    You type your orders out in natural language rather than choose options from a menu, and the ‘misinterpretation’ aspect turns a flaky parser from a bug to a feature.

    (I’d post links, but I seem to be having real problems posting anything at all so I simplified.)

  25. So, step one: find the enemy. Step two try to get him to accept battle on ground favorable to yourself. Step three, present your plan of battle to your council of subordinates and advisors and make sure everybody is onboard and understands what they are to do. Step four, array your army for battle.
    All of this to be done on limited information and lots of guesswork in full knowledge that no plan survives contact with the enemy.
    Sounds like oodles of fun.

  26. “Alas, not an option for most pre-modern commanders.”
    I find the word “most” in that sentence deeply disturbing.

  27. The ancient DOS game Sword of the Samurai gets this right sort of by accident. Before a battle starts, you select one of three formations (basically, do you put the bulk of your force in the middle, on the flanks, or concentrated on one flank), and then all your units follow that plan. During the battle, you can issue RTS-style commands, but the keyboard controls are so incredibly slow (there’s no mouse support, you have to move a cursor with the arrow keys) that it’s impossible to micromanage the whole army. Instead, you mostly watch the battle happen and only issue orders when something important happens. Like if a unit is getting routed, maybe you send another one to reinforce it.

  28. All your writings have led me to the point of wanting to see a game that implements what it would really be like to participate in (or lead) a realistic battle (not necessarily in the pre-modern era, but that’s an option). Real people in real modern battles still exist in a physical location and have Mk 1 Eyeballs, even if they have some tools that let them quickly get reasonably accurate information at distances far beyond what the eyeball can do, but even in the modern era with the most sophisticated tools available, there’s still uncertainty about who’s where, what their status is, what they’re doing, and what their goals are—even among your own side!

    A game that truly implements what it’s like to run a pre-modern battle would essentially need to lock the camera in a first-person perspective, and give you information only via your eyes and ears (although obviously an NPC could hand you a piece of paper with writing on it that you would then read). There’d have to be minimal UI elements, e.g. some kind of meter to indicate things that the game cannot communicate otherwise, like how tired/hungry/wounded you are; interfaces for choosing what to communicate to others; and so on. You’d have to rely on reports to find out just about anything, including how the battle went! Time would be an issue, too, because if you end up in a situation where one side or the other keeps refusing battle, you can’t really afford to spend a month sitting at your computer killing time waiting for something to change, so there’d need to be a way to fast-forward until something happened.

    It would be very different from typical gameplay and might not be as much “fun” as games are supposed to be, but it would be a fascinating experience.

    1. I think the big thing you’d need for that is a relatively sophisticated AI. An actual army has a lot of subcommanders who can exercise some greater degree of control over their troops and react intelligently to changes in circumstance.

      For instance, in Total War games I frequently end up with units just standing around, possibly under fire, because I’m watching another part of the fight. Actual troops wouldn’t (usually) stand around in the open to be shot at; they’d advance or retreat or form tesudo or something. They might make disasterously bad choices, but they’d do something

      So to get the experience you need an AI that can both follow orders and make independant decisions, and ideally be able to handle both tactical brillance and disasterous screwups depending on the centurion in question.

      1. There’s a good scene in Against Three Lands by George Phillies where the day is saved when an officer, trained under a system of rigid command, briefly given more in a more flexible system, takes the initiative, much to the surprise of the person who trained him.

    2. From what little I’ve seen, this game in development isn’t exactly like that, but its called ‘Fire & Maneuver’, and it seems to try and develop a game system which takes these sorts of limitations into account, i.e. making orders delayed rather than instantaneous. I haven’t played it yet, and I also haven’t played a real time strategy game in years, and I’m also not a professional military historian like Dr. Devereaux, so I’m not really in any position to judge the ‘accuracy’ of these things.

  29. Dr. D,

    You mentioned both armies “offering” battle by marching out out of their camps and to the terrain where they would like to fight.

    What stops an enemy army which has already formed up from attacking when the other army breaks formation to go back to their camp/a different position once the intial offer is refused

    1. This is in fact a concern! At Pydna, the day before the battle, Aemilius is quite careful one he decides not to fight, to pull his maniples out one by one so that he can get the fortified camp set up without rendering his army vulnerable to sudden attack.

      Once the fortified camp was set up, the defensive advantage it offered, combined with screening forces covering the army moving back into camp, is what is going to discourage a sudden assault. For two armies even roughly evenly matched, the defensive advantage of a fortified camp on a hill made any direct assault a very poor idea.

  30. After reading about all the uncertainty and lack of control, I can understand the importance of pre-battle religious rituals like casting for omens and making offerings. It must’ve truly felt like much was simply up to the fates.

  31. Interestingly, much of the Total War model of generalship—perfect information, instant communications, even the overhead perspective—goes back to *chess*.

    1. And chess is a pretty poor predictor of battlefield performance. We have some of Naploen’s games recorded.

      https://www.chessgames.com/perl/chessgame?gid=2013286
      https://www.chessgames.com/perl/chessgame?gid=1250611
      https://www.chessgames.com/perl/chessgame?gid=1250611
      https://www.chessgames.com/perl/chessgame?gid=1250610

      I’m only FIDE expert, not even titled (So there are MUCH stronger players than myself), and I’m completely confident that if these games are representative of Bonaparte’s normal play, I could crush him, and easily at that. I don’t think highly of my chances commanding an early 19th century army against him though.

      1. Chess is a lovely game, but I will say that one popular culture trope I absolutely hate is the way chess gets used to signal either ‘strategist’ or ‘intelligent person’ as if all smart people are good at chess or that chess skills are perfectly transferable to other disciplines.

        Lovely game, but yes – not a solid predictor of skill in other areas.

        1. Fancy what a game of chess would be if all the chessmen had passions and intellects, more or less small and cunning; if you were not only uncertain about your adversary’s men, but a little uncertain also about your own; if your knight could shuffle himself on to a new square by the sly; if your bishop, at your castling, could wheedle your pawns out of their places; and if your pawns, hating you because they are pawns, could make away from their appointed posts that you might get checkmate on a sudden. You might be the longest-headed of deductive reasoners, and yet you might be beaten by your own pawns. You would be especially likely to be beaten, if you depended arrogantly on your mathematical imagination, and regarded your passionate pieces with contempt. Yet this imaginary chess is easy compared with the game a man has to play against his fellow-men with other fellow-men for his instruments.

          ― George Eliot

        2. The man of system, on the contrary, is apt to be very wise in his own conceit; and is often so enamoured with the supposed beauty of his own ideal plan of government, that he cannot suffer the smallest deviation from any part of it. He goes on to establish it completely and in all its parts, without any regard either to the great interests, or to the strong prejudices which may oppose it.

          He seems to imagine that he can arrange the different members of a great society with as much ease as the hand arranges the different pieces upon a chess-board. He does not consider that the pieces upon the chess-board have no other principle of motion besides that which the hand impresses upon them; but that, in the great chess-board of human society, every single piece has a principle of motion of its own, altogether different from that which the legislature might chuse to impress upon it. If those two principles coincide and act in the same direction, the game of human society will go on easily and harmoniously, and is very likely to be happy and successful. If they are opposite or different, the game will go on miserably, and the society must be at all times in the highest degree of disorder.

          ― Adam Smith

          1. Adam Smith was a very smart man.
            Another very smart man, Sir Terry Pratchett, said: People are the problem.

        3. Given that computer software can defeat the vast majority of human chess players, I think that there is very little correlation between chess and real world military command. 🙂

  32. There’s an older RTS game called Kohan that actually handles a few things in a more realistic way (although much else is the same as usual for the genre). In particular, it’s company-based, not unit-based, and it’s impossible to give orders to companies once they’ve engaged in combat (except for “rout” and “retreat”, the latter being only available as long as the company commander is alive and the company’s morale is high enough). So tactics involves locating enemy companies, maneuvering your companies to engage on favorable terrain with favorable timing, and occasionally doing advanced tactics like engaging briefly with a scout force, and then retreating toward a fortified position to draw the enemy into a trap. Which is tricky to pull off, and requires preparation, and a whole lot of things can go wrong.

    Also, formations are limited to line (for battle), skirmish (for scouting), column (for fast marching) and pressed column (for really fast marching), and the faster the formation, the more it degrades combat readiness, such that if you press march a company into a battle, it will almost certainly get slaughtered. Unless you’re attacking from the rear, or unless the enemy has lost enough morale that they’re almost ready to break, or unless you can get close enough to their spellcaster support to engage them in melee combat.

    Another thing this post reminded me of was the novel “Dragon”, by Steven Brust, wherein our hero temporarily joins the army because the opposing general insulted him, and one of his own friends is the general of his side. Simple magic is widespread, and advanced magic is not uncommon. The advanced stuff is mostly used for scrying and weather control: frequent rains during marches are a sign that the enemy’s weather wizards are doing a better job than yours. And is an additionally bad thing because there’s a small attrition rate, every rainy day, from individual soldiers brain-frying themselves from pushing their umbrella spells too long while they’re too tired. There’s also a great discussion of how equipment has changed over time due to the effect of magic. Direct defense trumps direct attack, so there’s not much of that. Metal armor went out as soon as a “heat metal” spell was devised, but leather and wood armor and shields are still a thing. Long-range archery is mostly useless due to wind control, but javelins are still useful. The magical version of muskets lasted until someone figured out a spell to detonate all of them at once.

  33. I think this look at pre-modern generalship can be a useful counterargument to all of the bemoaning about the bureaucracy and pen-pushing ‘decadence’ of modern militaries; given the chance, any pre-modern general would jump to have all of the advantages his contemporary counterparts have, even if that comes with annoying stacks of paperwork and hanging back in a headquarters far from the front, because now he can actually control events in a meaningful way and see what is going on instead of just crossing his fingers and guessing.

  34. I was pleased to see the mention of “Take Command – Manassas”, since I thought that game addresses some of challenges facing an actual historical general more than the Total War series (and would be relevant to this series of posts). Better yet are the successor games to “Take Command”, being Scourge of War: Gettysburg (and its add-ons) for American Civil War and Scourge of War: Waterloo (and its add-ons). The SOW visuals aren’t great (especially compared to TW), but the gameplay is more interesting in my view. The SOW games allow for lots of flexibility in difficulty settings; plus there are lots of available mods which can enhance the realism. When played on HITS (“hind in the saddle”) you can be at close to horseback height (as in TC-Manassas). As Dr. Devereaux notes, that can make for a much different battle than in the TW series with a God’s-eye view.

    But there are several other features in the SOW games which increase the realism (depending upon your realism-settings), and thus your frustration as a leader. The frustration is more realistic, in my view. Essentially, “friction” is added (is that enough for a Clausewitz’ drink? – ha ha). Please allow me to explain.

    In the first place, the battle space on most SOW maps is very large (often about 5 miles v 5 miles). Depending upon your game settings, you can be forced to search out the enemy (and vice-versa), and that can take lots of marching over the map to do so. You might locate the enemy, without realizing that you have only encountered a smaller detachment as opposed to the main enemy army. Sometimes, the situation is reversed. And it’s often difficult to tell one from the other due to the next factor. A key element of SOW games using HITS and other more realistic settings is that you never get to see the whole battle. If you are brigade commander, your battle area is relatively small, and you have only little idea of how the rest of the battle is going. Even if within your general battle-space, you may be limited in your view. Consequently, you might spend time running around the battlefield to be able to increase your visibility, and knowledge of the respective numbers and disposition of the combatants. But when doing so, however, your ability to command, rally, and support your various units are limited. Third, in the SOW games, you can decide whether to be a brigade, division, corps, or army commander. As a practical matter, the higher up in the battle order, the less you can control the actual fighting (that seems to be oxymoronic, but isn’t). If you are a bridge commander, you can order your regiments into a particular formation at a particular location, and can order a single regiment to a location. Based upon the unit’s experience, morale, casualties, etc., it will usually follow orders. Well, most of time; you’ll have some micro-managing to do from time to time. But if you are a divisional commander, you can order a brigade commander to go to a particular location (or take some tactical stance), and now the brigade commander’s own “personality” comes into play. A cautious brigadier general might refuse to go into harm’s way. On the other hand, an aggressive brigadier general might decide to leave the excellent defensive position you selected for his brigade and your orders to hold that position, and instead go attack a more numerous enemy formation. You can never be sure that your brigadier general will follow your directive, and, even if so, it might only be in part. As a divisional commander, you have several brigades to command, so you can’t afford to micro-manage any of them. You can give some generalized orders to subordinates, but for the most part you have to trust them to do the right thing. As a corps or army commander, these “control” problems increase exponentially since “personalities” abound amongst many subordinate units. Fourth, you can also require couriers for orders to units of a particular size. As such, if you are brigade commander, a mounted courier leaves your location, travels to the unit receiving the order (often by nearby roads as opposed to straight-line), gives the orders, and then the unit gives orders to its component units (by courier as well depending upon the type of unit). Use of couriers thus adds a time delay to your orders. Furthermore, use of couriers can also add some more randomness in whether orders are received, because couriers can be killed if they get too close to enemy units while en route (this is a particular problem when you are a corps or army commander since some subordinate units aren’t in visible range; you might not realize your couriers keep getting killed by an enemy unit located in-between so your subordinate isn’t receiving your orders). Couriers can also be sent “upstairs” to your superior commanders, and your superiors may or may not decide to heed your request … as in reality, too.

    There are more features that can augment the levels of “friction” (especially depending upon your mods and settings). The friction can be very frustrating, but you can get a greater (but, of course, not equivalent) sense of the practical difficulties such commanders would have faced, and consequently a greater sense of accomplishment for a job well done.

  35. This also reminded me of the to-me-classic piece, “The Ultimate War Simulation Game”, by David Wong.

    https://web.archive.org/web/20071107054536/http://www.cracked.com/article_15660_ultimate-war-simulation-game.html

    I want a war sim …

    1. … where I spend two hours pushing across a map to destroy a “nuclear missile silo,” only to find out after the fact that it was just a missile-themed orphanage.

    I want little celebrities to show up on the scene and do interviews over video of charred teddy bears, decrying my unilateral attack. I want congressional hearings demanding answers to these atrocities.

    2. On the very next level I want to lose half of my units because another “orphanage” turned out to be an enemy ambush site. I want another round of hearings asking why I didn’t level that orphanage as soon as I saw it, including tearful testimony from a slain soldier’s daughter who is now, ironically, an orphan.

    3. Every war sim has a “Fog of War” that obscures the map in darkness until units scout the landscape. Well, I want a hazy, brown “Fog of Bullshit” layer below that. I want it to make a village of farmers look like a secret armed militia, I want it to show me a massive enemy fortress where there is actually an aspirin factory. I want to never know for sure which it was, even after the game is over.

  36. “So frequently final preparations for a battle were made with both armies encamped fairly close to each other, often separated by just a few miles of open field. ”

    This reminds me a bit of the North Sea in WW1. Two fleets, based in unreachable anchorages, facing each other, occasionally leaving port only with the intent of tempting the enemy to fight in conditions favourable to the tempter. Of course, ancient armies could not hope to arrange a submarine ambush of each other, but OTOH, the land is much less homogenous than the sea.

    Presumably, the goal of each side would be to get the other side to fight ion ground favourable to oneself. Which suggests that the army offering battle would need to look not just for ground favourable to itself, but for ground which would look very much less favourable to itself, when seen from the viewpoint of the enemy. Only then will they chose to attack.

    After all, if both sides decide to fight at a given time and place, the loser must have been mistaken to decide to fight there. So it is important to find some way of getting the enemy to make a worse decision than oneself.

    1. Sometimes the losing side had the advantage, but lost because of the fortunes of war. Of course this still requires explaining why a side would choose to fight at a disadvantage. Professor Devereaux will probably go into more detail later in the series, but he already mentioned one instance. “he Spartans subsequently lured the Argives into less favorable terrain by damaging the irrigation system, forcing a battle on their terms, which the Spartans won”. The alternative to battle is having your crops ravaged or city besieged or some higher campaign objective.

      1. Yep, battle is always a risk, but that goes both ways: If you’re at a disadvantag eand going to lose anyway, wagering batlte (even if you’re in a worse position) might be better than slowly losing without a chance to reverse things.

  37. Question about the establishing camps bit; if the attacker is exhausted and probably strung out from marching when they arrive in effective battle range, isn’t it in the interest of the defender to launch an immediate assault while they’re starting to encamp? If they’ve been holding position in their camp, relying on reports and knowledge of the terrain to determine the enemy’s path, they’d be fully rested up and the camps end up being in range for an army to attack within the day. Is it because it’s getting late and they don’t want to chance a night battle where their already limited control falls apart completely?

    1. Night battles are chancy – but the enemy has scouts out who report your position. They look for favourable ground within range for battle but not too close, and take up a defensive formation there while the camp is being built. You also have to contend with uncertainty – their light forces push back your scouts, some elements of their main force appears – but is it all? Maybe it’s the bait to lure you out (Montrose did something like this in one of his battles). By the time you are certain that the main enemy is there, they have a solid position.

      1. On top of this:

        1) The enemy is tired after a day’s march. Often, so are you, because you had to maneuver into the enemy’s path on purpose. Very rarely will Army A just rush at Army B while Army B has the luxury of sitting still in their camp and resting their feet all day, then taking a short stroll to hit Army A while only Army A is tired. And if Army B takes such an approach, they expose themselves to the risk that Army A will keep going and find some terrain position that gives them an advantage that’s risky in the hands of even a tired enemy. Such as now sitting astride Army B’s line of retreat.

        2) The enemy is tired, but how tired? Fatigue can be hard to quantify and can depend heavily on details like just how far the enemy marched, how many hills and mudholes and river crossings they had to deal with, how well nourished they are, and so on. An army that marched twenty miles up and down hills all day may well be so exhausted they can barely stand (but realistically, they’d have stopped and made camp 5-8 miles ago, leaving them effectively out of striking distance of your army anyway). An army that marched ten miles along a reasonably good road may be in good enough shape to leave you looking very stupid for ordering a night attack.

    2. Well, if the enemy shows up late, after a day of marching, you still have to form up your troops for battle. This gives the enemy time to put up a camp. And in any case, has our host said, you still would already nee to have an battle plan ready, and discussed with all your subordinates, to get an fast attack done.

      Great if you can pull it off. But a good way to loose a sizeable part of your army, if you need 3 hours to get everyone into gear, and than your left flank is in the wrong place, because your idiot brother in law, who leads them misunderstood your intentions.

  38. It’s worth noting that the more spread-out TWW2 formation from PvP play is in large part down to the need to limit the impact of magical attacks that deal damage across a large area. I imagine real-life historical formations would have been very different if they had to contend with Wind of Death and Final Transmutation.

    1. I suspect there’s probably a tipping point where you just don’t have close order heavy infantry and settled states start fighting more like nomads, with mages and smallish attached escorts launching hit-and-run raids either by stealth or entirely on horseback.

      Then there’s a big intermediate area that the Warhammer factions occupy where you don’t have enough mages around to totally reorient your fighting style around them but you do have to deal with the fact that a hundred feet of your front line can just evaporate every so often. So you need the ability to rapidly reinforce before the enemy can really exploit their breakthroughs. Also, you want snipers. This may be technologically difficult, of course.

      1. also, unlike in the TW:W video games, on the tabletop all armies have tools to directly cancel enemy spells (either using an item called scroll of dispel or with dice generated by one’s wizards), which would presumably be reflected in the lore. But I get why it’s not done in the game, as it doesn’t seem it would have made for a very fun gameplay.

        1. I had the idea once of a setting where magic was mostly ineffective in battle, because defensive/negating magic tended to win out, but if one side had decisive enough superiority to break through it became a battle-winning terror.

      2. It depends on the tech level you’re dealing with. Against an early modern army, it might just force them into something like the modern system of tactics, with concealment and avoiding bombardment being your main concerns. With a Medieval or ancient era military… Well OGH has made the point repeatedly that you can’t just reorient into a new system of warfare on the fly. If mages are just that destructive, maybe your battles are just small groups of mages getting into duels, and whoever wins the duel is understood to have won the entire battle.

        War by champion!

  39. How do you feel about the difference between Field Of Glory and Total War? Obviously neither is perfect and both have upsides and downsides. Total War is generally more popular for a variety of reasons including being real time.

  40. Bret, have you considered a post Part Zero: raising your army? A big reason for the “matching up” aspect of generalship in tabletop and computer games is because you have a lot of choice over what units make up your army.
    Maybe worth discussing how much control ancient generals in various states had over which troops were assigned to them?

    (Wait, there’s no zero in Roman numerals. Needs a new title.)

    1. The Romans themselves didn’t use zero, but in the middle ages people started using N (short for nulla) as the Roman numeral for zero.

  41. At some point I read about a game that dealt with fog of war and communication time in an interesting way, but I couldn’t find it again now. IIRC it was sci-fi, and each player could see the entire board and had a number of fleets with admirals. But each admiral had a rating (from 1 to 9 or something) and their moves were set in stone for the next rating number of turns. So for example for an admiral with rating 3 you would already have planned their next 3 moves and each turn you would add a 4th move to that queue. It was meant to represent the skill of the leadership in determining what was a good plan based on limited info. Not perfect (your best admirals can mind-read your worse ones from lightyears away for example), but interesting for sure.

    Another similar game which may fit the way armies look for another is Scotland Yard. In that game, one player is a “fugitive” and the rest are detectives, looking for him. The map consists of many crossings all over central London, and players move by taxi (short range), bus (medium range) and metro (sparse but high distance) across the city. The goal of the detectives is to reach the field the fugitive is on. The fugitive can only use move cards the detectives have used and must keep track of his moves by writing them down. Occasionally (I forgot what the mechanism for it was) the fugitive must place a token on a square he is/has been (again, I don’t know exactly) as if reports reach Scotland Yard about his whereabouts, helping them in their search. It seems a lot like how scouts look for the enemy army, except you have much less ability to use rumours and again there is no false information.

  42. “That distinction matters because technological changes after 1450 begin to alter some of these constraints, although generally slower than you might think.”

    Which technological changes are you thinking of? Portable looking-glass are one thing, but only affect information-gathering, not the dispatch of orders or the reception of reports. The first thing that I can think of would be telegraphs, but that’s post-1700 at the very least for optical telegraphs, and afaik, those were entirely static

    1. I’m not sure how relevant it would be to the fighting of the battle itself, but one important (and often underappreciated) change was the development of accurate maps, which meant that it was now possible to come up with detailed campaign plans in advance and to make a reasonable guess as to how long each army or part of the army would take to move from A to B (though obviously factors like bad weather or enemy action could still throw things off). A strategy like Napoleon’s, of marching the army in multiple semi-independent corps and then uniting them for the big decisive battle, wouldn’t really have been feasible in the medieval period, for example.

      1. Or often quite a bit later on, I remember hearing about Washington’s overly complicated battle strategies that relied on things that like and the problems that they ran into.

    2. For dispatching orders and reports, it is sufficient to have looking glasses, a pre-arranged code, signal flags or even dancers in a sufficiently visible position, and some means of getting the attention of the person holding the looking glass on the other end. Once contact has been made, with each end having a looking glass on the other end’s signal person, they can send arbitrary text messages at a rate of a character every few seconds.

      As far as I know, this kind of hand-signal system only emerged in the nineteenth century, even though the components had all been invented by the fifteenth.

      1. Yes, that’s Semaphores/optical telegraphy but these weren’t used for civilian, long distance, along pre-determined paths of towers, until late 18th century.

      2. On the other hand, Sun Tzu (two drinks!) said “Control of many is like control of a few; it is simply a matter of setting up signals in advance” several thousand years ago.

        I honestly wouldn’t call anything before WW2 modern communications-wise. Until you have portable radio, tactical control is by shouting and runners (and situationally, smoke, signal rockets, flags, musical instruments, pigeons etc) most of which were old when recorded history began.

        Telegraphs did improve strategic/operational level control to a good extent, although I’m not sure how it compares with the Persian, Roman or Mongol relay systems.

        1. The telegraph massively improved strategic / operational control in comparison with the Persian, Roman, or Mongol systems. Even the best organised horse relay system has a finite speed per day. The further your message needs to go, the more you multiply the delay.

          The telegraph isn’t quite immune to distance effects, but for most purposes you can send a message a thousand kilometres in about the same time as ten kilometres. Even when the distance was significant, as in the early trans-Atlantic cables, it was still taking just hours to transmit a page of text instead of weeks. Telegraph cables are also much less likely to be slowed down by heat, snow, wind, etc than messengers.

          There’s a great book “The Victorian Internet” by Tom Standage about how the telegraph revolutionised communications in the 19th C. (Complete with the in period complaints about junk telegrams, decay of morals, etc, etc.)

          While the telegraph wasn’t used for tactical control (although WW1 troops on the western front set up telephone lines) the much better strategic and operational comms in turn made the tactical problems easier to think about, as generals had a much better idea of where and when everybody was.

        2. One thing about telegraphs is that you could get quick confirmation that a message got through, which was huge. Telegraph lines could be and were severed but then at least you knew that the line was down. Dispatching a horse and rider was a shot in the dark.

      3. I’m now just imagining the highly-trained Army Corps of Signal Dancers, responsible for transmitting orders across long ranges with elaborate and flashy dance moves…

      4. Correction:
        I said “invented by the fifteenth [century].”
        I should have said “invented by the end of the fifteenth [century].”

  43. Reports still tended to be of dubious quality in WW2. In the pacific both sides tended to over report the amount and class of enemies sunk. Although the allies seemed to get it under controll somewhat in the late war. But in the early days of the war, it was very common that the report would read two cruisers and a battle ship sunk, when actually one destroyer was sunk, and another destroyer and one cruiser was damaged.

    Also I had some fun experience about getting in formation last weekend. I was on an LARP-convetion, and the NPC was almost double the size we usualy were. (Witch meas 100 instead of 50 people) Getting people from a coulmn to a line formation, is pretty easy, left person walks to the left, right person walks to the right whenever the last person in a unit left the coloumn they shout “through”, so you just have to count the numbers of “through”s to now when you have reached your final place in the line.
    It’s a frigging nightmare, to get people into a coloumn, in a way that everyone is, where they are supossed to be. It always takes twice as long, as it should. We had to split the heavy armored people (lovingly called plate-pigs) into two teams, so they would get into formation in time. Also had to double the amount of NCO-roles because of that.
    We also had the dubious fun of experiencing having to offer battle several times, when the other side refused, as some of the better organised player parties would put on gear, and form up, so we also had to get ready, but than all the other players refused to move out, so the players stood down. Not good for moral.

    1. The Battle off Samar went as well as it did for the Americans because the Japanese were continually misidentifying the ships as larger and armored. First meant that they got the range wrong, second that armor piercing shells would go entirely through the ship before exploding.

    2. On the LARPing, another piece of experimental archeology that confirms yes the Macedonian era pike phalanxes really did need all those NCOs!

      Although the Pacific War shows that the *accuracy* of reports from people is still dubious, it is a good demonstration of how modern-era communications have changed the way generals – OK, in this case admirals – carry out their tasks. WW2 admirals tended to receive too many reports rather than too few, and needed specialist staff to sort through the incoming flood and identify the crucial reports fast enough to be of use to the admiral.

      Even in the gigantic Pacific Ocean both Japanese and US admirals mostly – not always, but mostly – had a pretty good idea of where the enemy fleets were from radio direction finding, signals traffic analysis, and code breaking. Hence the Japanese in particular trying complex diversions and split formations in an attempt to generate tactical surprise.

      A pre-modern admiral wouldn’t have done so, firstly because the enemy scouting system probably wouldn’t have been good enough to fall for it, and secondly because they might not be able to find their own ships again in a reasonable time frame! It was common for high ranking naval commands and governments in the pre-modern era to not know where their own fleets were, let alone the enemy.

      1. Hence why something as simple as “send a portion of your army in a roundabout move to flank the enemy” like at Wittstock was so risky: It means you’re effectively unable to communicate with them while they’re moving and so you don’t know if they’re delayed, get intercepeted, etc.

  44. “In editions prior to 3rd (original game, early basic, AD&D1, BEMCI, AD&D2, 3.0, and 3.5 high level magic most definitely completely dominated over non-magical characters.”

    AIUI, 3rd edition was the peak of caster dominance, or “quadratic wizard, linear fighter”. While many individual spells were kind of nerfed, overall casters became stronger. Concentration checks meant they didn’t automatically lose a spell if hit, and the saving throw system was radically changed, so that even good saving throws wouldn’t keep up with the difficulties, let alone a classes’ bad saving throws. Plus, *so many* spells got defined, especially with supplements, so there was almost always a spell for some problem. Also, the highly detailed skills system, plus fighters getting few skill points, meant fighters had little to do other than fight. Full casters didn’t get many skill points either, but didn’t need them to be effective.

    By contrast, in BECMI getting hit fizzled your spells, and saving throws were absolute, not relative: a high level fighter (or halfing/dwarf) could shrug off most effects. And doing things other than fight or cast spells was more a matter of player skill and imagination than of character definition.

  45. The General can kinda get around his inability to have a bird’s eye view of the battlefield by building a mobile tower with himself at the top.

    Or be situated on a Mountaintop or a hill.

    He doesn’t have to remain on the ground on his horse.

    1. A mobile tower isn’t going to be very mobile unless your enemy is kind enough to fight you on . Read previous posts about sieges and siege technology: siege towers usually required the besiegers to level the ground along the approach path first and they could be stopped by a shallow ditch. Oh, and do you really want your general to be the tallest, most obvious target for anyone with a bow, crossbow, ballista?

      Being on the top of a mountain or a hill is almost certainly placing the general much further away from the battle, as big groups of infantry, let alone cavalry, need flat ground.

      Without fast detailed communication technology (I assume the subject of future posts in this series) a general might be able to see more from such a height, but they won’t be able to *do* much more with the extra information. A general at ground level is generally more useful, whether that is to lead the key attack in person (Alexander the Great) or reinforcing weak points in the line.

  46. What was the major incentive for deployment in extremely long, relatively thin lines? It definitely screws up command and control. Is there no bonus to concentration? I get that most combat was melee, still, wouldn’t it be much better to drive the entirety of one’s army into the enemy’s center? If their wings come around and attack from the flank or rear (if they can pull that off, which isn’t certain), you can deploy a part of your own army to check them, with the advantage of interior lines of communication as it were. Meanwhile, you’ve got a (brief or not) time when all your mass is concentrated against a portion of theirs.

    1. A large and complicated subject, so only a brief response. For more detail, I suggest playing some tabletop ancients / medieval wargames and trying out the various formations. (For reasons Bret is currently and has previously explained, the computer wargames are often not at all realistic.)

      Briefly, weapons and tactics.

      Missile weapons almost demand thin lines, because contrary to Hollywood which loves clouds of arrows arcing up high and down again, bows and javelins work best when firing with an almost flat trajectory. Which is difficult if there’s someone standing in front of you. So Roman legionaries with pila and English longbowmen both use shallower lines to get more people throwing / shooting at once.

      The most effective infantry melee tactic is usually a charge, giving your troops momentum as they physically crash into the enemy. A deep formation will usually be more effective, either because long pikes can be used by ranks behind the first, soldiers can physically lean into the backs of those in front of them, or just because a deep formation makes the soldiers in the front rank more confident and the enemy more scared. So yes there are plenty of deep formations in ancient and medieval warfare.

      BUT if your deep formation gets hit in the flank, or even threatened, by the the enemy wings, you’re in trouble. For reasons Bret will no doubt go into in future posts it’s hard to redeploy. The charging troops may be so focussed on what’s in front of them that they don’t notice the enemy on their flanks until it’s too late. Or if they do want to change facing they have to stop first, losing the momentum against the enemy to the front. Either scenario is bad, so better to deploy in a thinner line that has less momentum but less likely to get outflanked.

      BUT there are special cases where a deep formation works anyway, when the battle can be won by defeating one key part of the enemy army… it’s complicated.

      Cavalry generally fight in wide shallow lines. Horses are much longer than humans, so a second or subsequent rank of cavalry can’t reach the enemy with their weapons; and horses can’t physically support the horse in front of them the way infantry can.

  47. “Of course there are games that are entirely about strategy or tactics, but even in fantasy or pre-modern RPGs it is most common for the player’s character to be put in command of the Big Battle than to merely be a foot soldier in it”

    In my experience the most common trope is for the player character to nominally be in charge of the army but, for some inexplicable reason, fighting on the front lines like a common footmen.

    I understand it is for gameplay purposes, but it still strikes me as odd.

    1. This will probably come up later, but for certain periods it was absolutely expected for leaders to fight in the frontline (partially because once the reserves were committed, there wasn’t really much left for them to *do* because ofa ll the control problems, so going in and inspiring the men was as good a use of the general as anything else)

  48. “This leads to situations where generals often know what general region an enemy force is in and they may have a sense of its ultimate objective (because the report they have was that the army was in such-and-such town moving down such-and-such road ten days ago) but often not its exact position at any given time.”

    Ha! Reminds of the Uncertainty Principle from quantum mechanics: the total uncertainty in the position and momentum of a (pre-modern) army (enemy or friendly!) can ultimately only be reduced to what a Mark I Eyeball can see from any single given location.

    This discussion reminded of the 1998 game Battlezone, which—despite being about a fantasy alt-cold war fought throughout the Solar System between the US and USSR using hover tanks made of living metal of alien origin (it’s complicated!)—had an interesting model where it was sort of a fusion of a first-person shooter and a real-time strategy game. Battles were fought between tanks*, each of which contained a pilot, who would eject when the tank got destroyed. You, yourself, despite being in command of everything on your side, were also such a pilot, and would also get ejected (and thus be supremely vulnerable when you got back down to ground level if you hadn’t managed to navigate away from any active enemy units). You could also command individual units (though for, I’m sure, both performance and players’ sakes with the UI you were limited to a total of ~30 units at one time), and they’d try to follow your orders, engaging with enemy units as they came across them. And due to your position in the cockpit of a tank, you often couldn’t tell exactly where where enemy units were if they weren’t directly in your line of sight (like, if they were behind a hill).

    However, you could also build (later in the game, due to the expense) satellite towers, which would allow you to access a more traditional RTS-style overhead view where you could use your mouse to select units and give them orders (your current vehicle excepted), though you still couldn’t see enemy units (other than vague radar pings) if they weren’t in line of sight of one of your units. Obviously, it was still very little like pre-modern generalship (you had instant and uninterruptible radio communication for relaying your orders, for instance, although due to the UI getting units to do things outside your LOS could range from tricky to impossible without the top-down view), but it was certainly interesting trying to command combat engagements (even very small ones) from “on the ground,” where you generally didn’t have much more information about, say, possible enemy reinforcements beyond what you could personally see (and the assistance of the 360° radar map in your cockpit to give you spatial awareness, though it had a limited range and didn’t handle intervening hills well).

    *At least, that’s what the game called them, though I’m less sure now after the recent series. Though they mainly focused on direct fire the primary (i.e., not “Light” or “Rocket”) version also allowed you to mount a ‘mortar’ weapon which could allow for some mean indirect fire ambushes.

Leave a Reply to guyCancel reply